Download as pdf or txt
Download as pdf or txt
You are on page 1of 69

Best Approach

Functions
(Sheet)

By Mathematics Wizard
Manoj Chauhan Sir (IIT Delhi)
No. 1 Faculty on Unacademy,
Exp. More than 14 Years in
Maths IIT-JEE ‘Best Approach’ (MC SIR) Functions

KEY CONCEPTS (FUNCTIONS)


THINGS TO REMEMBER
1. GENERAL DEFINITION
If to every value (Considered as real unless otherwise stated) of a variable x, which belongs to some
collection (Set) E, there corresponds one and only one finite value of the quantity y, then y is said to be
a function (Single valued) of x or a dependent variable defined on the set E ; x is the argument or
independent variable .
If to every value of x belonging to some set E there corresponds one or several values of the variable y,
then y is called a multiple valued function of x defined on E.Conventionally the word "FUNCTION” is
used only as the meaning of a single valued function, if not otherwise stated.
x f (x )  y
Pictorially : 
    , y is called the image of x & x is the pre-image of y under f.
input output

Every function from A  B satisfies the following conditions .


(i) f  Ax B (ii)  a  A  (a, f(a))  f and
(iii) (a, b)  f & (a, c)  f  b = c

2. DOMAIN, CODOMAIN & RANGE OF A FUNCTION :


Let f : A  B, then the set A is known as the domain of f & the set B is known as co-domain of f .
The set of all f images of elements of A is known as the range of f . Thus :
Domain of f = {a  a  A, (a, f(a))  f}
Range of f = {f(a)  a  A, f(a)  B}
It should be noted that range is a subset of codomain . If only the rule of function is given then the domain of
the function is the set of those real numbers, where function is defined. For a continuous function, the interval
from minimum to maximum value of a function gives the range.

3. IMPORTANT TYPES OF FUNCTIONS :


(i) POLYNOMIAL FUNCTION :
If a function f is defined by f (x) = a0 xn + a1 xn1 + a2 xn2 + ... + an1 x + an where n is a non negative
integer and a0, a1, a2, ..., an are real numbers and a0  0, then f is called a polynomial function of degree
n.
NOTE :
(a) A polynomial of degree one with no constant term is called an odd linear
function . i.e. f(x) = ax , a  0
(b) There are two polynomial functions , satisfying the relation ;
f(x).f(1/x) = f(x) + f(1/x) . They are :
(i) f(x) = xn + 1 & (ii) f(x) = 1  xn, where n is a positive integer.
Proof : Consider f (x) = a0 + a1x + a2x2 + ..... + anxn.
Equate f (x) × f (1/x) = f (x) + f (1/x) & equate coefficients.

(ii) ALGEBRAIC FUNCTION :


y is an algebraic function of x, if it is a function that satisfies an algebraic equation of the form
P0 (x) yn + P1 (x) yn1 + ....... + Pn1 (x) y + Pn (x) = 0 Where n is a positive integer and P0 (x), P1
(x) ........... are Polynomials in x. e.g. x3 + y3 – 3xy = 0 or
y = x is an algebraic function, since it satisfies the equation y²  x² = 0.
Note that all polynomial functions are Algebraic but not the converse. A function that is not algebraic is
called TRANSCEDENTAL FUNCTION .

Get 10% Instant Discount On Unacademy Plus [Use Referral Code: MCSIR] 2
Maths IIT-JEE ‘Best Approach’ (MC SIR) Functions
(iii) FRACTIONAL RATIONAL FUNCTION :
g(x )
A rational function is a function of the form. y = f (x) = , where
h (x )
g (x) & h (x) are polynomials & h (x)  0.

(IV) EXPONENTIAL FUNCTION :


A function f(x) = ax = ex ln a (a > 0 , a  1, x  R) is called an exponential function. The inverse of the
exponential function is called the logarithmic function .
i.e. g(x) = loga x.
For f (x) = ex domain is R and range is R+.
1
For f (x) = e x domain is R – {0} and range is R+ – {1}.
Note that f(x) & g(x) are inverse of each other & their graphs are as shown.

(v) ABSOLUTE VALUE FUNCTION :


A function y = f (x) = x is called the absolut e value function or Modulus function. It is defined as
 x if x  0
: y = x 
  x if x  0
For f (x) = | x |, domain is R and range is R+  {0}.
1
For f (x) = , domain is R – {0} and range is R+.
|x|

(vi) SIGNUM FUNCTION :


A function y= f (x) = Sgn (x) is defined as follows :

 1 for x  0
y = f (x) =  0 for x  0

  1 for x  0

It is also written as Sgn x = |x|/ x ;


x  0 ; f (0) = 0

(vii) GREATEST INTEGER OR STEP UP FUNCTION :


The function y = f (x) = [x] is called the greatest integer function where [x] denotes the greatest integer
less than or equal to x . Note that for :
1  x < 0 ; [x] =  1 0x< 1 ; [x] = 0
1x< 2 ; [x] = 1 2x < 3 ; [x] = 2
and so on .

Get 10% Instant Discount On Unacademy Plus [Use Referral Code: MCSIR] 3
Maths IIT-JEE ‘Best Approach’ (MC SIR) Functions
Properties of greatest integer function :
(a) [x]  x < [x] + 1 and
x  1 < [x]  x , 0  x  [x] < 1
(b) [x + m] = [x] + m if m is an integer .
(c) [x] + [y]  [x + y]  [x] + [y] + 1
(d) [x] + [ x] = 0 if x is an integer
=  1 otherwise .
(viii) FRACTIONAL PART FUNCTION :
It is defined as :
g (x) = {x} = x  [x] .
e.g. the fractional part of the no. 2.1 is
2.1 2 = 0.1 and the fractional part of  3.7 is 0.3.
The period of this function is 1 and graph of this function
is as shown .

4. DOMAINS AND RANGES OF COMMON FUNCTION :

Function Domain Range


(y = f (x) ) (i.e. values taken by x) (i.e. values taken by f (x) )

A. Algebraic Functions

(i) xn , (n  N) R = (set of real numbers) R, if n is odd


R+  {0} , if n is even
1
(ii) 
x n , (n N) R – {0} R – {0} , if n is odd

R+ , if n is even

(iii) x1 / n , (n  N) R, if n is odd R, if n is odd


+ +
R  {0} , if n is even R  {0} , if n is even

1
(iv) 1/ n , (n  N) R – {0} , if n is odd R – {0} , if n is odd
x
R+ , if n is even R+ , if n is even

B. Trigonometric Functions
(i) sin x R [–1, + 1]
(ii) cos x R [–1, + 1]

(iii) tan x R – (2k + 1) , kI R
2

(iv) sec x R – (2k + 1) , kI (–  , – 1 ]  [ 1 ,  )
2
(v) cosec x R – k , k  I (–  , – 1 ]  [ 1 ,  )
(vi) cot x R – k , k  I R

Get 10% Instant Discount On Unacademy Plus [Use Referral Code: MCSIR] 4
Maths IIT-JEE ‘Best Approach’ (MC SIR) Functions
C. Inverse Circular Functions (Refer after Inverse is taught )

  
(i) sin–1 x [–1, + 1]  2 , 2 
 
(ii) cos–1 x [–1, + 1] [ 0, ]
  
(iii) tan–1 x R  , 
 2 2

  
(iv) cosec –1x (–  , – 1 ]  [ 1 ,  )  2 , 2  – { 0 }
 

 
(v) sec–1 x (–  , – 1 ]  [ 1 ,  ) [ 0, ] –  
2 
(vi) cot –1 x R ( 0, )

Function Domain Range


(y = f (x) ) (i.e. values taken by x) (i.e. values taken by f (x) )

D. Exponential Functions
(i) ex R R+
(ii) e1/x R–{0} R+ – { 1 }
(iii) ax , a > 0 R R+
(iv) a1/x , a > 0 R –{0} R+ – { 1 }

E. Logarithmic Functions

(i) logax , (a > 0 ) (a  1) R+ R


1
(ii) logxa = log x R+ – { 1 } R–{0}
a

(a > 0 ) (a  1)

F. Integral Part Functions Functions


(i) [x] R I
1 1 
(ii) R – [0, 1 )  , n  I  {0} 
[x ] n 

G. Fractional Part Functions

(i) {x} R [0, 1)


1
(ii) R–I (1, )
{x}

Get 10% Instant Discount On Unacademy Plus [Use Referral Code: MCSIR] 5
Maths IIT-JEE ‘Best Approach’ (MC SIR) Functions
H. Modulus Functions
(i) |x| R R+  { 0 }
1
(ii) |x| R–{0} R+

I. Signum Function

|x|
sgn (x) = ,x0 R {–1, 0 , 1}
x
=0,x=0

J. Constant Function

say f (x) = c R {c}

5. EQUAL OR IDENTICAL FUNCTION :


Two functions f & g are said to be equal if :
(i) The domain of f = the domain of g.
(ii) The range of f = the range of g and
(iii) f(x) = g(x) , for every x belonging to their common domain. eg.
1 x
f(x) = & g(x) = 2 are identical functions .
x x
6. CLASSIFICATION OF FUNCTIONS :
One  One Function (Injective mapping) :
A function f : A  B is said to be a oneone function or injective mapping if different elements of A
have different f images in B . Thus for x1, x2  A & f(x1) ,
f(x2)  B , f(x1) = f(x2)  x1 = x2 or x1  x2  f(x1)  f(x2) .
Diagramatically an injective mapping can be shown as

OR

Note : (i) Any function which is entirely increasing or decreasing in whole domain, then
f(x) is oneone .
(ii) If any line parallel to xaxis cuts the graph of the function atmost at one point,
then the function is oneone .
Many–one function :
A function f : A  B is said to be a many one function if two or more elements of A have the same
f image in B . Thus f : A  B is many one if for ; x1, x2  A , f(x1) = f(x2) but x1  x2 .
Diagramatically a many one mapping can be shown as

OR

Get 10% Instant Discount On Unacademy Plus [Use Referral Code: MCSIR] 6
Maths IIT-JEE ‘Best Approach’ (MC SIR) Functions
Note : (i) Any continuous function which has atleast one local maximum or local minimum,then f(x) is
manyone . In other words, if a line parallel to xaxis cuts the graph of the function atleast
at two points, then f is manyone .
(ii) If a function is oneone, it cannot be manyone and vice versa .
Onto function (Surjective mapping) :
If the function f : A  B is such that each element in B (codomain) is the f image of atleast one element
in A, then we say that f is a function of A 'onto' B . Thus f : A  B is surjective iff  b  B,  some
a  A such that f (a) = b .
Diagramatically surjective mapping can be shown as

OR

Note that : if range = codomain, then f(x) is onto.


Into function :
If f : A  B is such that there exists atleast one element in codomain which is not the image of any
element in domain, then f(x) is into .
Diagramatically into function can be shown as

OR

Note that : If a function is onto, it cannot be into and vice versa . A polynomial of degree even will
always be into.
Thus a function can be one of these four types :

(a) oneone onto (injective & surjective)

(b) oneone into (injective but not surjective)

(c) manyone onto (surjective but not injective)

(d) manyone into (neither surjective nor injective)

Note : (i) If f is both injective & surjective, then it is called a Bijective mapping.
The bijective functions are also named as invertible, non singular or biuniform functions.
(ii) If a set A contains n distinct elements then the number of different functions defined from
A  A is nn & out of it n ! are one one.
Identity function :
The function f : A  A defined by f(x) = x  x  A is called the identity of A and is denoted by IA.
It is easy to observe that identity function is a bijection .
Constant function :
A function f : A  B is said to be a constant function if every element of A has the same f image in B .
Thus f : A  B ; f(x) = c ,  x  A , c  B is a constant function. Note that the range of a constant
function is a singleton and a constant function may be one-one or many-one, onto or into .

Get 10% Instant Discount On Unacademy Plus [Use Referral Code: MCSIR] 7
Maths IIT-JEE ‘Best Approach’ (MC SIR) Functions
7. ALGEBRAIC OPERATIONS ON FUNCTIONS :
If f & g are real valued functions of x with domain set A, B respectively, then both f & g are defined in
A  B. Now we define f + g , f  g , (f . g) & (f/g) as follows :
(i) (f ± g) (x) = f(x) ± g(x)
(ii) (f  g) (x) = f(x)  g(x)
f f (x)
(iii)   (x) = domain is {x  x  A  B s . t g(x)  0} .
 g g ( x)

8. COMPOSITE OF UNIFORMLY & NON-UNIFORMLY DEFINED FUNCTIONS :


Let f : A  B & g : B  C be two functions. Then the function gof : A  C defined by
(gof) (x) = g (f(x))  x  A is called the composite of the two functions f & g .
x f (x)
Diagramatically      g (f(x)) .
Thus the image of every x  A under the function gof is the gimage of the fimage of x .
Note that gof is defined only if  x  A, f(x) is an element of the domain of g so that we can take its g-image.
Hence for the product gof of two functions f & g, the range of f must be a subset of the domain of g.
PROPERTIES OF COMPOSITE FUNCTIONS :
(i) The composite of functions is not commutative i.e. gof  fog .
(ii) The composite of functions is associative i.e. if f, g, h are three functions such that fo (goh) &
(fog) oh are defined, then fo (goh) = (fog) oh .
(iii) The composite of two bijections is a bijection i.e. if f & g are two bijections such that gof is
defined, then gof is also a bijection.
9. HOMOGENEOUS FUNCTIONS :
A function is said to be homogeneous with respect to any set of variables when each of its terms
is of the same degree with respect to those variables .
For example 5 x2 + 3 y2  xy is homogeneous in x & y . Symbolically if ,
f (tx , ty) = tn . f (x , y) then f (x , y) is homogeneous function of degree n .

10. BOUNDED FUNCTION :


A function is said to be bounded if f(x)  M , where M is a finite quantity .

11. IMPLICIT & EXPLICIT FUNCTION :


A function defined by an equation not solved for the dependent variable is called an
IMPLICIT FUNCTION . For eg. the equation x3 + y3 = 1 defines y as an implicit function. If y has been
expressed in terms of x alone then it is called an EXPLICIT FUNCTION.
12. INVERSE OF A FUNCTION :
Let f : A  B be a oneone & onto function, then their exists a unique function
g : B  A such that f(x) = y  g(y) = x,  x  A & y  B . Then g is said to be inverse of f . Thus
g = f1 : B  A = {(f(x), x)  (x, f(x))  f} .
PROPERTIES OF INVERSE FUNCTION :
(i) The inverse of a bijection is unique .
(ii) If f : A  B is a bijection & g : B  A is the inverse of f, then fog = IB and
gof = IA , where IA & IB are identity functions on the sets A & B respectively.
Note that the graphs of f & g are the mirror images of each other in the
line y = x . As shown in the figure given below a point (x ',y ' ) corresponding to y = x2 (x >0)
changes to (y ',x ' ) corresponding to y   x , the changed form of x = y .

Get 10% Instant Discount On Unacademy Plus [Use Referral Code: MCSIR] 8
Maths IIT-JEE ‘Best Approach’ (MC SIR) Functions

(iii) The inverse of a bijection is also a bijection .


(iv) If f & g are two bijections f : A  B , g : B  C then the inverse of gof exists and
(gof)1 = f1 o g1 .
13. ODD & EVEN FUNCTIONS :
If f (x) = f (x) for all x in the domain of ‘f’ then f is said to be an even function.
e.g. f (x) = cos x ; g (x) = x² + 3 .
If f (x) = f (x) for all x in the domain of ‘f’ then f is said to be an odd function.
e.g. f (x) = sin x ; g (x) = x3 + x .
NOTE : (a) f (x)  f (x) = 0 => f (x) is even & f (x) + f (x) = 0 => f (x) is odd .
(b) A function may neither be odd nor even .
(c) Inverse of an even function is not defined .
(d) Every even function is symmetric about the yaxis & every odd function is symmetric about the
origin.
(e) Every function can be expressed as the sum of an even & an odd function.
f ( x )  f (  x ) f ( x)  f (  x)
e.g. f ( x)  
2 2

(f) The only function which is defined on the entire number line & is even and odd at the same time
is f(x) = 0.
(g) If f and g both are even or both are odd then the function f.g will be even but if any one of
them is odd then f.g will be odd .
14. PERIODIC FUNCTION :
A function f(x) is called periodic if there exists a positive number T (T > 0) called the period of the
function such that f (x + T) = f(x), for all values of x within the domain of x.
e.g. The function sin x & cos x both are periodic over 2 & tan x is periodic over .
NOTE : (a) f (T) = f (0) = f (T) , where ‘T’ is the period .
(b) Inverse of a periodic function does not exist .
(c) Every constant function is always periodic, with no fundamental period.
(d) If f (x) has a period T & g (x) also has a period T then it does not mean that
f (x) + g (x) must have a period T . e.g. f (x) = sinx+ cosx.
1
(e) If f(x) has a period p, then and f (x) also has a period p.
f (x)
(f) if f(x) has a period T then f(ax + b) has a period T/a (a > 0).
15. GENERAL :
If x, y are independent variables, then :
(i) f(xy) = f(x) + f(y)  f(x) = k ln x or f(x) = 0.
(ii) f(xy) = f(x) . f(y)  f(x) = xn, n  R
(iii) f(x + y) = f(x) . f(y)  f(x) = akx.
(iv) f(x + y) = f(x) + f(y)  f(x) = kx, where k is a constant.

Get 10% Instant Discount On Unacademy Plus [Use Referral Code: MCSIR] 9
Maths IIT-JEE ‘Best Approach’ (MC SIR) Functions
Solved Examples :
1. Solve the equation |2x – 1| = 3[x] + 2{x} where [.] denotes greatest integer and {.} denotes fractional
part function
Sol. We are given that, |2x – 1| = 3[x] + 2{x}
1
Let, 2x – 1  0 i.e. x  . The given equation yields.
2
1 – 2x = 3[x] + 2{x}
1  5[x]
 1  2[x]  2{x}  3[x]  2{x}  1  5[x]  4{x}  {x} 
4
1  5[x] 3 1
 0  1  0  1  5[x]  4    [x] 
4 5 5
3 1
Now, [x] = 0 as zero is the only integer lying between  and
5 5
1 1 1 1
 {x}   x  which is less than . Hence is one solution.
4 4 2 4
1
Now, let 2x  1  0 i.e. x 
2
 2x – 1 = 3[x] + 2{x}  2[x] + 2{x} – 1 = 3[x] + 2{x}
1
 [x] = –1  – 1  x < 0 which is not a solution as x 
2
1
 x is the only solution.
4

2. The number of solution of the equation sgn({x}) = |1 – x| where {} denotes fractional part function is
(A) 4 (B) 3 (C) 2 (D) 1
Sol. sgn ({x}) = |1 – x| has solution only at x = 1.

y = |x – 1 |
(0,1)
y = sgn{x}

x
–1 0 1 2 3

3. Let S = {1, 2, 3, 4}. Then number of functions f : S  S such that f(k)  2 k for all k  S, is
(A) 32 (B) 64 (C) 128 (D) 156
Sol. No. of mapping = 2 × 4 × 4 × 4 = 128

Get 10% Instant Discount On Unacademy Plus [Use Referral Code: MCSIR] 10
Maths IIT-JEE ‘Best Approach’ (MC SIR) Functions
1
4. Find the domain f (x)  where [.] denotes greatest integer function.
| [| x | 5] | 11
Sol. |[|x| – 5]|| – 11
so [|x| – 5] > 11 or [|x| – 5] < –11
[|x|] > 16 [|x|] < –6
|x|  17 or |x| < –6 (Not possible)
 x  – 17 or x  17
so x  (– , –17]  [17, )
x2  x  c 5 3
5. If the range of function f (x)  2
, x  R is  ,  then c is equal to
x  2x  c 6 2
(A) –4 (B) 3 (C) 4 (D) 5
x2  x  c
Sol. Let y 
x 2  2x  c
 (y – 1) x2 + (2y – 1) x + c(y – 1) = 0
As x is real, so D  0
 (2y – 1)2  4c(y – 1)2
 4(c – 1)y2 + 4(1 – 2c) y + (4c – 1)  0 ....(1)
But we are given
(6y – 5) (2y – 3)  0  12y2 – 28y + 15  0 ....(2)
c  1 1  2c
 On comparing (1) and (2), we get 
3 7
4c  1
 c4
15
6. Find the number of integers in the range of the function f (x)  cos x(sin x  sin 2 (x)  3) .

Sol. Let y  cos x(sin x  sin 2 x  3)


 (y sec y – sin x)2 = sin2x + 3
 y2 sec2x + sin2x – 2y tan x = sin2x + 3
 y2 + y2 + tan2x – 2y tan x – 3 = 0
 y2 tan2x – 2y tan x + (y2 – 3) = 0
As tan x is real.
 D0
 4y2 – 4y2(y2 – 3)  0
 4 – y2  0
 y2 – 4  0
 –2y2
Ans is 5

7. The graph of y = g(x) is shown in the adjacent figure. If mi (i = 1, 2, 3, ....... , n) are the integral values
of m for which the equation (g(x))2 – (2m + 5)g(x) + (10 – 2m) = 0 has exactly six solutions then find the

 n 
  mi 
value of  i1 .
 n 1 
 
 
Get 10% Instant Discount On Unacademy Plus [Use Referral Code: MCSIR] 11
Maths IIT-JEE ‘Best Approach’ (MC SIR) Functions

g(x)

x
0 1 2

Sol. Let g(x) = t


Consider,

O 3

f(t) = t2 – (2m + 5)t + 10 – 2m


Case-I : When  = 0 and 0 <  < 3
f(0) = 0, f(3) > 0

 On taking intersection, m 


Case-II : When  > 3 and 0 <  < 3
f(0) > 0 and f(3) < 0

Get 10% Instant Discount On Unacademy Plus [Use Referral Code: MCSIR] 12
Maths IIT-JEE ‘Best Approach’ (MC SIR) Functions

1 
 m   ,5 
2 
 Number of integral values of m is {1, 2, 3, 4} i.e., 4 values, m1 = 1, m2 = 2, m3 = 3, m4 = 4

 n 
  mi  10
  i1  2
 n 1  5
 
 

x 2  3x  a
8. Let f (x)  2 , where f : R  R. Find the value of parameter 'a' so that the given function is
x  x 1
one-one.
x 2  3x  a
Sol. f (x) 
x2  x 1

(x 2  x  1)(2x  3)  (x 2  3x  a)(2x  1) 2x 2  2x(1  a)  (3  a)


f '(x)  
(x 2  x  1) 2 (x 2  x  1) 2
Let, g(x) = – 2x2 + 2x (1 – a) + (3 – a)
g(x) will be negative if 4(1 – a)2 + 8(3 – a) < 0
 1 + a2 – 2a + 6 – 2a < 0  (a – 2)2 + 3 < 0
which is not possible. Therefore function is not monotonic.
Hence, no value of a is possible.

9. If the function f(x) and g(x) are defined on R  R such that

 x  3, x  rational  x  5, x  irrational
f (x)   and g(x)   x  rational
 4x, x  irrational   x,
then (f – g) (x) is
(A) one-one and onto (B) neither one-one nor onto
(C) one-one but not onto (D) onto but not one-one
Sol. We have (f – g) (x) = (f(x) – g(x))
 2x  3, x  rational

3x  5, x  irrrational

Get 10% Instant Discount On Unacademy Plus [Use Referral Code: MCSIR] 13
Maths IIT-JEE ‘Best Approach’ (MC SIR) Functions

 3   5
As f    0  f  
 2   3 
and so on.
 f(x) is many one function.
Also,  5 does not belong to the range, because if
3x  5   5
 x  0Q
f(x) is into function
Ans. is B

10. Number of surjective function f : A  B, where set A contains 4 elements and set B contains 3 elements,
is
(A) 36 (B) 40 (C) 44 (D) 56
4 3 4 3
Sol. 3 – [ C1(2 – 2) + C2] = 36

n 1
 2 , if n is odd
11. Let f : N  N be defined as f (n)  
n , if n is even
 2
(A) injective but not surjective (B) surjective but not injective
(C) both injective as well as surjective (D) neither injective nor surjective
Sol. As f(1) = 1 = f(2)
f(3) = 2 = f(4)
So, f is not injective
Also, Rf = N, so f is surjective,
Hence f is surjective but not injective.

12. Which of the following pair(s) of function have same graphs ?


sec x tan x cos x sin x
(A) f (x)   , g(x)  
cos x cot x sec x cos ecx
 2 2  
(B) f(x) = sgn(x2 – 4x + 5), g(x)  sgn  cos x  sin  x    where sgn denotes signum function
  3 
2 3x 3)
(C) f (x)  e ln(x , g(x)  x 2  3x  3

sin x cos x 2 cos 2


(D) f (x)   , g(x) 
sec x cos ecx cot x
sec x tan x cos x sin x
Sol. (A) We have f (x)   , g(x)  
cos x cot x sec x cos ecx
k
Clearly both f(x) and g(x) are identical functions as x   kI
2
(B) As x2 – 4x + 5 = (x – 2)2 + 1 > 0
Hence f(x) = 1  x  R

Get 10% Instant Discount On Unacademy Plus [Use Referral Code: MCSIR] 14
Maths IIT-JEE ‘Best Approach’ (MC SIR) Functions

2 2 
Also cos x  sin  x    0
 3
Hence g(x) = 1  x  R
 f(x) and g(x) are identical
3 3x 3)
(C) f (x)  eln(x

2
2  3 3
As x  3x  3   x     x  R
 2 4
Hence f(x) = x2 + 3x + 3  x  R
 f(x) is identical to g(x)

sin x cos x 2 cos 2 x


(D) We have f (x)   , g(x) 
sec x cos ecx cot x
k
Clearly both f(x) and g(x) are identical functions as x   kI
2

1
 4 1 7 7
13. If g(x)   4 cos (x)  2cos(2x)  cos 4(x)  x  , then the value of g(g(100)) is equal to
 2 
(A) –1 (B) 0 (C) 1 (D) 100
1
Sol. We have 4 cos 4 x  2 cos 2x  cos 4x  x 7
2
1
 4 cos 4 x  2(2 cos 2 x  1)  (2 cos 2 2x  1)  x 7
2
1
3
We get g(x)    x 7 
7

2 
1 1
3 7  3  3 7
 g(g(x))    (g(x)) 7       x 7    x
2  2 2 
Ans. is D

14. Let f(x) = ax + b, where a and b are integers. If f(f(0)) = 0 and f(f(f(4))) = 9, then the value off(f(f(f(10))))
is equal to
(A) 0 (B) 4 (C) 9 (D) 10
Sol. f(x) = ax + b
f(0) = b
f(f(0)) = a.b + b = 0  (a + 1) b = 0  a = – 1 or b = 0
f(f(f(4))) = 9
ff(f(4a + b)) = 9
 f(a(4a + b) + b) = 9
 f(4a2 + ab + b) = 9
 a(4a2 + ab + b) + b) = 9
 4a3 + a2b + ab + b = 9

Get 10% Instant Discount On Unacademy Plus [Use Referral Code: MCSIR] 15
Maths IIT-JEE ‘Best Approach’ (MC SIR) Functions

3 9
where b = 0, 4a3 = 9  a   a is not an integer
4
when a = –1, –4 + b – b + b = 9  b = 13
 f(x) = – x + 13
f(f(x)) = – (–x + 13) + 13 = x
 f(f(f(f(10)))) = f(f(10)) = 10

 1 
15. Let f (x)    where [y] and {y} denote greatest integer and fractional part function respectively
 cos{x} 
and g(x) = 2x2 – 3x(k + 1) + k(3k + 1). If g(f(x)) < 0  x  R then find the number of integral values of
k.
Sol. f(x) = 1  x  R
 g(1) < 0  x  R
 2 – 3(k + 1) + 3k2 + k < 0
 3k2 – 2k –1 < 0
 (3k + 1) (k – 1) < 0
 1 
 k   ,1
 3 
Ans. (1)

16. Composition of piecewise defined functions :


If f(x) = | |x – 3| – 2| 0x4
g(x) = 4 – |2 – x| –1  x  3
then find fog(x) and draw rough sketch of fog(x).
Sol. f(x) = | | x – 3 | – 2 | 0  x  4

1  x 0  x  1
| x  1| 0  x  3 
  x  1 1  x  3
| x  5 | 3  x  4 5  x 3  x  4

g(x) = 4 – |2 – x| –1  x  3
4  (2  x)  1  x  2 2  x  1  x  2
 
4  (x  2) 2  x  3 6  x 2 x 3

1  (2  x) 0  2  x  1 and  1  x  2
2  x  1 1  2  x  3 and  1  x  2
1  g(x) 0  g(x)  1 
 5  (2  x) 3  2  x  4 and  1  x  2
 fog(x)  g(x)  1 1  g(x)  3  
5  g(x) 1 6  x 0  6  x  1 and 2x3
 3  g(x)  4 
6  x  1 1  6  x  3 and 2x3

5  6  x 3  6  x  4 and 2x3

Get 10% Instant Discount On Unacademy Plus [Use Referral Code: MCSIR] 16
Maths IIT-JEE ‘Best Approach’ (MC SIR) Functions

1  x  2  x  1 and  1  x  2 1  x  2  x  1 and  1  x  2


1  x 1  x  1 and  1  x  2 1  x 1  x  1 and  1  x  2

3  x 1 x  2 and  1  x  2 3  x 1 x  2 and  1  x  2
 
x  5  6   x  5 and 2  x  3 x  5 5 x 6 and 2  x  3
5  x  5   x  3 and 2  x  3 5  x 3 x 5 and 2  x  3
 
x 1  3  x  2 and 2  x  3 x  1 2x3 and 2  x  3

1  x  1  x  1

 3  x 1  x  2
x  1 2  x  3

f(g(x))
2
1

x
–1 1 2 3

x|x|
17. If f (x)  then f–1 (x) is
1 x2
|x| x
(A) ; 1  x  1 (B) ; 1  x  1
1 | x | 1 x

x |x|
(C) sgn(x). ; 1  x  1 (D) sgn(x). ; 1  x  1
1 x 1 | x |

y
Sol. x ; x  0, 0  y  1
1 y

y
 ; x  0, 1  y  0
1 y
Ans. (D)

Get 10% Instant Discount On Unacademy Plus [Use Referral Code: MCSIR] 17
Maths IIT-JEE ‘Best Approach’ (MC SIR) Functions
18. (a) Let f(x) = x2 + 2x ; x  –1. Draw graph of f–1(x) also find the number of solutions of the equation,
f(x) = f–1(x)
(b) If y = f(x) = x2 – 3x + 1, x  2. Find the value of g'(1) where g is inverse of f
y
y = f(x)
x = –1

y=f–1(x)
Sol. (a) x
–2 –1 0
(–1,–1)

f(x) = f–1(x) is equivalent to f(x) = x


 x2 + 2x = x  x(x + 1) = 0  x = 0, – 1
Hence two soution for f(x) = f–1(x)
(b) y=1  x2 – 3x + 1 = 1
 x(x – 3) = 0  x = 0, 3
But x2  x=3
Now g(f(x)) = x
Differentiating both sides w.r.t x
1
 g '(f (x)).f '(x)  1  g '(f (x)) 
f '(x)
1 1 1
 g '(f (3))   g '(1)   (As f '(x)  2x  3)
f '(3) 63 3
Alternate Method
y = x2 – 3x + 1 ; x2 – 3x + 1 – y = 0
3  9  4(1  y) 3  5  4y
x 
2 2
3  5  4y
x  2; x 
2
3  5  4x 1
g(x)  ; g '(x)  0  4
2 4 5  4x
1 1 1
g '(1)   
54 9 3

1
19. Let f, g : R  R be defined by f(x) = 3x – 1 + |2x + 1| and g(x)  ((3x  5)  | 2x  5 |) , then
5
(A) fog = gof
(B) (fog)–1 = gog
1 1
(C) y = min (fog(x), (fog(x))2, (fog(x))3, ......(fog(x))101) then at x  , y is 2011
2 2

fogofogofog....fog (5)  3
(D)   
100 times

Get 10% Instant Discount On Unacademy Plus [Use Referral Code: MCSIR] 18
Maths IIT-JEE ‘Best Approach’ (MC SIR) Functions

 1  5
 x  2 x  2  x  2 x  2
Sol. f (x)   , g(x)  
5x 1 x 5
x x
 2  5 2

 f(x) and g(x) are inverse of each other.


 y = min {fog(x), (fog(x))2 (fog(x))3, ......(fog(x))2011}
 y = min {x, x2, x3, ..... x2011}
1 1
 y    2011
 2 2
Ans. is A,C

20. Let f(x) is an odd function defined on R such that f(1) = 2, f(3) = 5 and f(–5) = –1. The value of
f (f (f (3)))  f (f (0))
is
3f (1)  2f (3)  f (5)
2 2 2 2
(A) (B) (C) (D)
3 5 5 3
Sol. f(x) + f(–x) = 0 for an odd function
 Numerator = f(f(–f(3))) = f(f(–5))
= f(–1) = –f(1) = – 2
Denominator = 3(2) –2(5) + f(–5) = 6 – 10 – 1 = – 5
Ans is C

21. The smallest natural number k for which


 x2 
f (x)  n  x 3  x 6  1   sin 5x    is an odd function  x  [–2, 2], is ([y] denotes largest
k 
integer  y)
(A) 38 (B) 39 (C) 40 (D) 41
Sol. Since f(x) is an odd function and  x 3  x 6  1   sin 5x is an odd function.

 x2  x2
    0 x  [2, 2] 0  1 x  [2, 2]
k  k
Hence k > 42
 Smallest natural number k = 40

 4n  x 
x

 x  e  e 
 4n 4n 1  
x0
22. For n  N, the function g(x)   (x sgn x)
  is
 ex  e x 
  
 10, x0
(A) odd function (B) even function
(C) neither even nor odd function (D) constant function

Get 10% Instant Discount On Unacademy Plus [Use Referral Code: MCSIR] 19
Maths IIT-JEE ‘Best Approach’ (MC SIR) Functions
Sol. As g(–x) = g(x)  x  R
 g(x) is an even function
| sin 4x |  | cos 4x |
23. The period of is
| sin 4x  cos 4x |  | sin 4x  cos 4x |
  
(A) (B) (C) (D) 
4 2 8

Sol. Period of | sin 4x |  | cos 4x | is
8

Period of | sin 4x  cos 4x |  | sin 4x  cos 4x |
8

Because period of | sin x  cos x |  | sin x  cos x |
2

Period of given function is
8
24. Let f : R  R be defined by f(x) = max. (1 + |x|, 2 – |x|) then which of the following hold(s) good ?
(A) f is periodic function. (B) f is neither injective nor surjective.
3 
(C) f is even function (D) Range of f   ,  
2 
Sol. Graph of f(x)
y

y = 1 + |x|

(0,2)

(–1/2, 3/2) (1/2, 3/2)

(0,1)

y = 2 – |x|
x
O (0,0)
x = (–1/2) x = (1/2)

Ans. (B,C,D)

n
25. For x  where n  I, the range of function
2
f(x) = sgn(sin x) + sgn(cos x) + sgn(tan x) + sgn(cot x) is equal to
Note: sgn x denotes signum function of x.
(A) {–2, 4} (B) {–2, 0, 4} (C) {–4, –2, 0, 4} (D) {0, 2, 4}

Get 10% Instant Discount On Unacademy Plus [Use Referral Code: MCSIR] 20
Maths IIT-JEE ‘Best Approach’ (MC SIR) Functions
Sol. f(x) periodic with period 2.

 
4; 0x
2

2; 
x
 2
f (x)  
0; 3
 x
 2
 3
2  x  2
 2
 Range of function = {–2, 0, 4}

x
26. Find the sum of all solution of the equation cot  log 2 {x} in x  (0,100) .
2
[Note: {k} denotes the fractional part function of k.]
x
Sol. We plot the graph of y  cot and y  log 2{x}
2
x
y = cot
y 2

 1 3/2 2 3 7/2
x
4
–1
y=log2{x}

3 7 11
We can observe point of intersection are , , ,....
2 2 2
3 7 11 199
and the sum is    ....   2525
2 2 2 2

19
27. If f(x) is a function such that f(x – 1) + f(x + 1) = 3f (x) and f(5) = 100, then find  f (5  12r) .
r 0

Sol. Given, f(x – 1) + f(x + 1) = 3f (x) .....(1)


from (1), f(x+ 1) + f(x + 3) = 3f (x  2) .....(2)
adding (1) and (2)
f (x  1)  f (x  3)  2f (x  1)  3[f (x)  f (x  2)]
 3 . 3 f (x  1)
 f(x – 1) + f(x + 3) = f(x+ 1) .....(3)
 f(x + 1) + f(x + 5) = f(x+ 3) .....(4)
adding (3) and (4)
f(x – 1) + f(x + 1) + f(x + 3) + f(x + 5) = f(x + 1) + f(x + 3)
 f(x – 1) = –f(x + 5)
 f(x) = –f(x + 6) .....(5)

Get 10% Instant Discount On Unacademy Plus [Use Referral Code: MCSIR] 21
Maths IIT-JEE ‘Best Approach’ (MC SIR) Functions
 f(x + 12) = f(x)
19
Now,  f (5  12r) = f(5) + f(5 + 12) + f(5 + 2.12) + .....+f(5 + 19.12)
r 0
= f(5) + f(5) + f(5) + ...... upto 20 terms
= 20f(5) = 20 × 100 = 2000

 2f (x) 
28. If x  log 4   , then find (f(2010) + f(–2009))
 1  f (x) 
 2f (x) 
Sol. Given, x  log 4  1  f (x) 
 

2f (x)  4x 
  4 x  f (x)     f (1  x)  f (x)  1
1  f (x)  2  4x 
So, f(2010) + f(–2009) = 1

1 n
29. If f(x) is a polynomial of degree n such that f (0)  0, f (1)  ,......f (n)  , then find f(n + 1)
2 n 1
Sol. (x + 1) f(x) – x is a polynomial of degree n + 1
 (x + 1) f(x) – x = k(x) [x – 1] [x – 2] ..... [x – n]
 [n + 2] f(n + 1) – (n + 1) = k[(n + 1)!]
Also, 1 = k (–1) (–2) ..... ((–n + 1))
1 = k (–1)n+1 (n + 1)!
 (n + 2) f(n + 1) – (n + 1) = (–1)n+1  f(n + 1) = 1, is n is odd
n
and , if n is even.
n 1

1
30. If the relation af (x)  bf    g(x) , (x  0) does not provide a unique f(x) then prove that
x
1
g 2 (x)  g 2    0 .
x

Sol. 1 .....(1)


af (x)  bf    g(x)
x
1 1
 af    bf (x)  g   .....(2)
x x
(1) × a – (2) × b
2 1
 a f (x)  abf    ag(x)
x
1 1
ab f    b 2f (x)  bg  
x x

2 2 1
 (a  b ) f (x)  ag(x)  bg  
x
Get 10% Instant Discount On Unacademy Plus [Use Referral Code: MCSIR] 22
Maths IIT-JEE ‘Best Approach’ (MC SIR) Functions
Since it does not provid unique f(x)
1
a2 – b2 = 0 and ag(x)  bg    0
x
1
(a – b) (a + b) = 0 and ag(x)  bg    0
x
1
when a = b, g(x)  g    0
x
1
when a = –b, g(x)  g    0
x
1
 g 2 (x)  g 2    0
x

Get 10% Instant Discount On Unacademy Plus [Use Referral Code: MCSIR] 23
Maths IIT-JEE ‘Best Approach’ (MC SIR) Functions
FUNCTIONS
CONCEPT BUILDING-01
1. Which of the following is a function ?
(i) {(2, 1), (2, 2), (2, 3), (2, 4)} (ii) {(1, 4), (2, 5), (1, 6), (3, 9)}
(iii) {(1, 2), (3, 3), (2, 3), (1, 4)} (iv) {(1, 2), (2, 2), (3, 2), (4, 2)}
2. Find the domains of definitions of the following functions :
(Read the symbols [*] and {*} as greatest and fractional part functions respectively.)

(i) f(x) = cos 2x  16  x 2 (ii) f(x) = log7 log5 log3 log2(2x3 + 5x2 – 14x)

1  5x
(iii) f(x) = ln  2
x  5x  24  x  2  (iv) f(x) =
7 x  7

 2 log10 x  1 
(v) y = log10 sin(x – 3) + 16  x 2 (vi) f(x) = log100x  
 x 

1 x
(vii) f(x) = 2
+ ln x(x2 – 1) (viii) f(x) = log 1 2
4x  1 2
x 1

1 2 2
(ix) f(x) = x 2  | x |  2
(x) f(x) = (x  3x  10)  l n (x  3)
9x

cos x  (1 / 2)
(xi) f(x) = log x (cos 2x) (xii) f(x) =
6  35x  6x 2

[x]
(xiii) 
f(x) = log1/3 log 4 [x]2  5   (xiv) f(x) =
2x  [x]

(xv) f(x) = logx sin x

3. Find the domain & range of the following functions.

(Read the symbols [*] and {*} as greatest and fractional part functions respectively).

2x
(i) f(x) = log 5  2(sin x  cos x)  3  (ii) f(x) =
1  x2

x 2  3x  2 x
(iii) f(x) = (iv) f(x) =
x2  x  6 1 | x |

(v) f(x) = 2 + x – [x – 3] (vi) f(x) = log3 (5 + 4x – x2)

Get 10% Instant Discount On Unacademy Plus [Use Referral Code: MCSIR] 24
Maths IIT-JEE ‘Best Approach’ (MC SIR) Functions
4. The range of the function f(x) = |x – 1| + |x – 2|, –1  x  3 is

(A) [1, 3] (B) [1, 5] (C) [3, 5] (D) None of these

5. The range of the function f(x) = 2|sin x| – 3|cos x| is :

(A) [–2, 13 ] (B) [–2, 3] (C) [3, 13 ] (D) [–3, 2]

6. (i) The function f(x) is defined on the interval [0, 1]. Find the domain of definition of the functions.

(a) f(sin x) (b) f(2x + 3)

(ii) Given that y = f(x) is a function whose domain is [4, 7] and range is [–1, 9]. Find the range and domain of

1
(a) g(x) = f(x) (b) h(x) = f(x – 7)
3

CONCEPT BUILDING-02
1. Classify the following functions f(x) definzed in R  R as injective, surjective, both or none.
x2
(a) f(x) = (b) f(x) = x + |x|
1  x2
2 2
ex  e x
(c) f(x) = ex – e–x (d) f(x) = 2 2
e x  e x

 1  1
2. If f(x) = |x| and g(x) = [x], then value of fog    + gof    is
 4  4
(A) 0 (B) 1 (C) –1 (D) 1/4
3. If f : R  R, f(x) = x3 + 3, and g : R  R, g(x) = 2x + 1, then f–1og–1(23) equals :
(A) 2 (B) 3 (C) (14)1/3 (D) (15)1/3
4. Which of the following functions has its inverse :
(A) f : R  R, f(x) = ax (B) f : R  R, f(x) = |x| + |x – 1|
(C) f : R  R+, f(x) = |x| (D) f : [, 2]  [–1, 1], f(x) = cos x
 2, when x  Q
5. If function f(x) =  , (fof) 4 the value will be :
 0, when x  Q

(A) 0 (B) 2 (C) 2 (D) None of these

1 x   3x  x 3 
6. If f(x) = log   and g(x) =  1  3x 2  , then f[g(x)] is equal to :
 1 x   

(A) – f(x) (B) 3f(x) (C) [f(x)]3 (D) None of these


7. If f : R  R, g : R  R and f(x) = 3x + 4 and (gof) (x) = 2x – 1, then the value of g(x) is :
1
(A) 2x – 1 (B) 2x – 11 (C) (2x – 11) (D) None of these
3

Get 10% Instant Discount On Unacademy Plus [Use Referral Code: MCSIR] 25
Maths IIT-JEE ‘Best Approach’ (MC SIR) Functions
8. If f : R  R, f(x) = x2 + 2x – 3 and g : R  R, g(x) = 3x – 4, then the value of fog(x) is :
(A) 3x2 + 6x – 13 (B) 9x2 – 18x + 5 (C) (3x – 4)2 + 2x – 3 (D) None of these
   
9. If f(x) = sin2 x + sin2  x   + cos x cos  x   and g(x) is a one-one function defined in R  R, then
 3  3
(gof) (x) is
(A) one-one (B) onto
(C) constant function (D) periodic with fundamental period 
10. Compute the inverse of the functions :
x
10x  10 x

(a) f(x) = ln x  x 2  1  (b) f(x) = 2 x 1 (c) y =
10 x  10  x

CONCEPT BUILDING-03
1. Find whether the following function are even or odd or none

x(a x  1)
(a) 
f(x) = log x  1  x 2
 (b) f(x) =
a x 1
(c) f(x) = sin x + cos x

(d) f(x) = x sin2 x – x3 (e) f(x) = sin x – cos x

 1 1
2. Let f  x   = x2 + 2 (x  0), then f(x) equals :
 x x

(A) x2 – 2 (B) x2 – 1 (C) x2 (D) None of these


3. Find the period of following function :
(i) f(x) = |sin 2x| is :
(A) /4 (B) /2 (C)  (D)

 x   x 
(ii) f(x) = sin   + cos   is
 2   2 
(A) 4 (B) 6 (C) 12 (D) 24
(iii) f(x) = log cos 2x + tan 4x is
(A) /2 (B) (C) 2 (D) 2/5
4. In the following which function is not periodic
(A) tan 4x (B) cos 2x (C) cos x2 (D) cos2 x
5. Suppose f is a real function satisfying f(x + f(x)) = 4f(x) and f(1) = 4. Find the value of f(21).
6. Let 'f' be a function defined from R+  R+. If [f(xy)]2 = x(f(y))2 for all positive numbers x and y and
f(2) = 6, find the value of f(50).
7. Let f(x) be a function with two properties
(i) for any two real number x and y, f(x + y) = x + f(y) and
(ii) f(0) = 2
Find the value of f(100).

Get 10% Instant Discount On Unacademy Plus [Use Referral Code: MCSIR] 26
Maths IIT-JEE ‘Best Approach’ (MC SIR) Functions
8. The period of cos(x + 4x + 9x +....+ n2x) is /7, then n  N is equal to :
(A) 2 (B) 3 (C) 4 (D) 5
9. Write explicitly, functions of y defined by the following equations and also find the domains of definition
of the given implicit functions :
(a) 10x + 10y = 10 (b) x + |y| = 2y

 1
10. Function f & g are defined by f(x) = sin x, x  R ; g(x) = tan x, x  R –  K   
 2

where K  I. Find

(i) Periods of fog & gof (ii) Range of the function fog and gof

ASSIGNMENT (Prilepko)
FUNCTION (DOMAIN AND RANGE)
Find the domains of definition of the following functions (1 – 112) ?
1. y  2x  x 2
2. y  x 1 x 1
3. y  x 1  6  x
4. y  x 2  5x  6

x3
5. y
5 x
6. f (x)  2  x  1  x
7. y  4x 2  4x  3
8. y  6  7x  3x 2
1
9. y  2 x
x 1
1
10. y 2
2x  5x  3
11. f (x)  4x  x 3
12. f (x)  3x  x 3
1
13. y 3
x x2

4  3x  x 2
14. y
x4

Get 10% Instant Discount On Unacademy Plus [Use Referral Code: MCSIR] 27
Maths IIT-JEE ‘Best Approach’ (MC SIR) Functions

3x  7
15. y 6
x 1  2

12  x  x 2
16. f (x) 
x(x  2)

6
17. y  5 x 
x
18. f (x)  x 2  x  20  6  x

x2  x  6
19. f (x) 
x2  4

x  12  x 2
20. y
x2  9
4 x2
 1 1
21. y  
 2 x 1

17  15x  2x 2
22. y
x3

7x
23. y
2
4x  19x  12

x 2  7x  12
24. y
x 2  2x  3

x 2  5x  6
25. y
x 2  6x  8
26. y  x  x 2  3x  x 2  2
1
27. y  x 2  x  20 
2
x  5x  14
1
28. y  x 2  x  20
2
14  5x  x

x 4  3x 2  x  7
29. y 1
x 4  2x 2  1
1
30. f (x) 
sin x  cos 4 x
4

31. f (x) = arcsin 3x


32. f (x)  (sin x  cos x) 2  1

Get 10% Instant Discount On Unacademy Plus [Use Referral Code: MCSIR] 28
Maths IIT-JEE ‘Best Approach’ (MC SIR) Functions

1
cos x 
33. y 2
6  35x  6x 2

log3 (x 2  1)
34. y
sin 2 x  sin x  0.25
1
35. y
3  log 3 (x  3)

x5
36. y
log(9  x)

3log 64 x  1
37. f (x)  3
2x  11
x2
38. y  log 2
x2
x3
39. f (x)  log
x 1
40. y  log(x  1)

x 2  8x  7
41. y  log .
x2  7
42. y  1  x  log  x  1 .
43. y  x  1  log 1  x  .
44. y = log ((x2 – 3x) (x + 5)).
45. y  4x  x 2  log 3 (x  2).

46. y  x 2  4x  5  log(x  1).


47. f (x)  log(5x 2  8x  4)  x  1.
48. y  x 2  4x  5  log(x  5).

log(3  2x  x 2 )
49. y
x

3 x
50. y  log .
x

1  2x
51. y  log .
x 3

52. f (x)  4 x  x  log(x  2).

x 2  5x  6
53. y
log(x  10)2
Get 10% Instant Discount On Unacademy Plus [Use Referral Code: MCSIR] 29
Maths IIT-JEE ‘Best Approach’ (MC SIR) Functions
log x
54. y .
2
x  2x  63

5x  x 2
55. y  log .
4

56. y  (x 2  3x  10) log 2 (x  3).

57. f (x)  log(1  4  x 2 ).


58. y  log(5x 2  8x  4)  (x  3)0.5 .

1  5x
59. y .
7x  7
60. y  4x  x 2  log(x 2  1).

4x
61. y  1  log(x  1)  .
x2

x 1
62. y  log 0.3 .
x5

63. y  log 0.4 (x  x 2 )

64. y  log 0.3 (x 2  5x  7).

65. y  log 0.5 (x 2  9)  4

x 1 1
66. y  log 0.4  2 .
x  5 x  36
1
67. f (x)  log 0.5 (  x 2  x  6)  2
.
x  2x

 log 0.3 (x  1)
68. y .
 x 2  2x  8

69. f (x)  16x  x 5  log 1 (x 2  1).


2

x
70. y  log 1 2
.
2
x 1

3x 2 18x  29
71. f (x)  4 x3
 26x 17 .

72. y  log 0.5 (3x  8)  log 0.5 (x 2  4).

73. f (x)  4x  x 3  log(x 2  1).

Get 10% Instant Discount On Unacademy Plus [Use Referral Code: MCSIR] 30
Maths IIT-JEE ‘Best Approach’ (MC SIR) Functions

1
74. y 4 log 4 16  log 8 (x 2  4x  3).
2

 2 x  1
75. f (x)  log 4  2  4 x  
 x 2

3x  4 x
76. y .
2x 2  x  8

  6  
77. f (x)  log 2   log 1  1  4   2 .
 2
 x 

6x  x 2  5
78. y .
5x  2  1
x
79. y .
x 2  5x  6
80. y   x 2  2x  3  log 3 (x  1)
x
81. y  log – x – 3.
x–2

x 2 – 2x
82. f (x)  .
log5 (x –1)
83. f (x) = log2x–5 (x2 – 3x – 10).
2
6 ( x– 2)
84. f (x)  4  8 x 3
– 52 – 22(x –1) .
1
 2 – f ' (x)  2
85. y  log1.7  ,
 x  1 

1 3 3 2 3
where f (x)  x – x – 2x  .
3 2 2
log 0.3 | x – 2 |
86. y .
|x|

87. y  6 x  x 2 – 2x 3 .
x
88. y x–4–  log (39 – x).
x–5
89. y = log (1 – log (x2 – 5x + 16)).
 3x – 1 
90. y  log 0.5  – log 2 .
 3x  2 

91. y  log log x – log (4 – log x) – log 3.

Get 10% Instant Discount On Unacademy Plus [Use Referral Code: MCSIR] 31
Maths IIT-JEE ‘Best Approach’ (MC SIR) Functions

92. y  log x – 2 (x 2 – 8x  15) .

93. y  log  8–2  log x – 3 42–log x 


2 log x  1
94. y  log100x .
–x
  1  
95. y  log 2  – log 1  1  4  – 1 .
 2
 x 
96. y = log|x| – 4 2.
97. y  sin x  16 – x 2 .
98. y = log (log2 x – 5 log x + 6)
x 1
99. y  log 1
2
3x  5

100. y  log sin(x  3)  16  x 2


log x
101. y
x 2  2x  63
x 3
102. y  arcsin  log(4  x)
2
3  2x
103. y  3  x  arcsin
5
x 2  2x  3
log (0.5  x )
4x 2  4x  3
104. y  (x  0.5)
2 log x  2
105. y  log100x
x
2x  1
106. y  arccos
2 2x
2
107. y  arccos
2  sin x
108. y  3sin x  1

x
109. y  2sin
2
1
110. y
4cos x  1
111. y  2 cos 2 x  3cos x  1
112. y  sin 2 x  sin x

Get 10% Instant Discount On Unacademy Plus [Use Referral Code: MCSIR] 32
Maths IIT-JEE ‘Best Approach’ (MC SIR) Functions
Find the domains of definition and the ranges of the following functions (113 – 120).
x
113. y
|x|

114. f (x)  x  x 2
115. y  3x 2  4x  5
116. y = log (3x2 – 4x + 5)
117. y = log (5x2 – 8x + 4)
118. f (x)  x  1  2 3  x

sin x  cos x  3 2
119. f (x)  log 2
2
120. f (x)  2  x  1  x

EXERCISE–I
1. Find the domains of definitions of the following functions :
(Read the symbols [*] and {*} as greatest integers and fractional part functions respectively.)

  
  log 1  1 
(i) f (x) = log2  1/ 2
 + log10 log10 x   log10  4  log10 x   log10 3



x
sin 100   

1 1 1
(ii) f (x) = + log1 – {x}(x2 – 3x + 10) + +
[x ] 2| x| sec(sin x)

1
 7 
(iii) f (x) = (5x  6  x ) lnx +
2
(7 x  5  2x ) +  ln  2  x  
2
  

(iv) f (x) = log  1


x 2  x  6  16 x C 2 x 1  203x
P2 x 5
 x  x 

 3 
(v) f (x) = log10  log |sin x| ( x 2  8x  23)  

 log 2 | sin x | 

2. Find the domain & range of the following functions.


(Read the symbols [*] and {*} as greatest integers and fractional part functions respectively.)

(i) y = 2  x  1  x

(ii) f (x) = log(cosec x - 1) (2  [sin x]  [sin x]2)

Get 10% Instant Discount On Unacademy Plus [Use Referral Code: MCSIR] 33
Maths IIT-JEE ‘Best Approach’ (MC SIR) Functions

x 4 3
(iii) f (x) =
x 5

3. (a) Draw graphs of the following function, where [ ] denotes the greatest integer function.

(i) f (x) = x + [x]

(ii) y = (x)[x] where x = [x] + (x) & x > 0 & x  3

(iii) y = sgn [x]

(iv) sgn (x x)

(b) Identify the pair(s) of functions which are identical ?


(where [x] denotes greatest integer and {x} denotes fractional part function)

(i) f (x) = sgn (x2 – 3x + 4) and g (x) = e[{x}]

1  cos 2 x
(ii) f (x) = and g (x) = tan x
1  cos 2 x

(iii) f (x) = ln(1 + x) + ln(1 – x) and g (x) = ln(1 – x2)

cos x 1 sin x
(iv) f (x) = and g (x) =
1  sin x cos x

4. Classify the following functions f(x) definzed in R  R as injective, surjective, both or none .
x 2  4x  30
(a) f(x) = (b) f(x) = x3  6 x2 + 11x  6 (c) f(x) = (x2 + x + 5) (x2 + x  3)
x 2  8x  18

5. Solve the following problems from (a) to (e) on functional equation.


(a) The function f (x) defined on the real numbers has the property that
f  f ( x ) ·1  f ( x )  = – f (x) for all x in the domain of f. If the number 3 is
in the domain and range of f, compute the value of f (3).

(b) Suppose f is a real function satisfying f (x + f (x)) = 4 f (x) and f (1) = 4. Find the value of f (21).
(c) Let 'f' be a function defined from R+  R+ . If [ f (xy)]2 = x  f ( y) 2 for all positive numbers x and y and
f (2) = 6, find the value of f (50).
(d) Let f (x) be a function with two properties
(i) for any two real number x and y, f (x + y) = x + f (y) and (ii) f (0) = 2.
Find the value of f (100).

Get 10% Instant Discount On Unacademy Plus [Use Referral Code: MCSIR] 34
Maths IIT-JEE ‘Best Approach’ (MC SIR) Functions

(e) Let f be a function such that f (3) = 1 and f (3x) = x + f (3x – 3) for all x.

Then find the value of f (300).

ax 8  bx 6  cx 4  dx 2  15x  1
(f) Suppose that f (x) is a function of the form f (x) =
x
(x  0). If f (5) = 2 then find the value of f (– 5).

6. Suppose f (x) = sin x and g (x) = 1 – x . Then find the domain and range of the following functions.
(a) fog (b) gof (c) fof (d) gog

    5
7. If f(x) = sin²x + sin²  x    cos x cos x   and g    1 , then find (gof) (x).
 3  3 4

 1 x 
8. A function f : R  R is such that f   = x for all x  – 1. Prove the following.
1 x 
(a) f  f ( x )  = x (b) f 1 x  = – f (x), x  0 (c) f (– x – 2) = – f (x) – 2.

x
9. (a) Find the formula for the function fogoh, given f (x) = ; g (x) = x10 and h (x) = x + 3. Find also the
x 1
domain of this function. Also compute (fogoh)(–1).
(b) Given F (x) = cos2(x + 9). Find the function f, g, h such that F = fogoh.

10. If f (x) = max  x, 1 x  for x > 0 where max (a, b) denotes the greater of the two

real numbers a and b. Define the function g(x) = f (x) · f 1 x  and plot its graph.

11. (a) The function f (x) has the property that for each real number x in its domain, 1/x is also in its domain and
f (x) + f 1 x  = x. Find the largest set of real numbers that can be in the domain of f (x)?

(b) Let f (x) = ax 2  bx . Find the set of real values of 'a' for which there is at least one
positive real value of 'b' for which the domain of f & the range of f are the same set.

 x if x 1
 1  x if x  0 
12. f (x) =  2 and g (x) =  1  x find (fog)(x) and (gof)(x)
 x if x  0 if x 1
13. Find whether the following functions are even or odd or none
2

(a) f(x) =
1  2  x

2x
x x
(b) f(x)= x
 1
e 1 2

Get 10% Instant Discount On Unacademy Plus [Use Referral Code: MCSIR] 35
Maths IIT-JEE ‘Best Approach’ (MC SIR) Functions

(c) f(x) = [(x+1)²]1/3 + [(x 1)²]1/3

 x 2n  e1/x  e 1/x  
(d) f(x) =  2n 2n 1  1/x 1/x   , x 0 and n  N
 (x sgn x)  e  e  

14.(i) Write explicitly, functions of y defined by the following equations and also find the domains of definition
of the given implicit functions :
(a) 10x + 10y = 10 (b) x + y= 2y

(ii) The function f(x) is defined on the interval [0,1]. Find the domain of definition of the functions.
(a) f (sin x) (b) f (2x+3)

(iii) Given that y = f (x) is a function whose domain is [4, 7] and range is [–1, 9]. Find the range and domain of
1
(a) g (x) = f (x) (b) h (x) = f (x – 7)
3

log10 x
15. Find the inverse of f (x) = 2  8 and hence solve the equation f (x) = f–1(x).

16.(a) Suppose that f is an even, periodic function with period 2, and that f (x) = x for all x in the interval
[0, 1]. Find the value of f (3.14).

(b) Find out for what integral values of n the number 3 is a period of the function:
f(x) = cos nx . sin (5/n) x.

17. Let f(x) = ln x and g(x) = x2 – 1


Column-I contains composite functions and column-II contains their domain. Match the entries of column-
I with their corresponding answer is column-II.
Column-I Column-II
(A) fog (P) (1, )
(B) gof (Q) (–, )
(C) fof (R) (–, –1)  (1, )
(D) gog (S) (0, )

18. The graph of the function y = f (x) is as follows.

Match the function mentioned in Column-I with the respective graph given in Column-II.

Get 10% Instant Discount On Unacademy Plus [Use Referral Code: MCSIR] 36
Maths IIT-JEE ‘Best Approach’ (MC SIR) Functions
Column-I Column-II
y
1
–2 –1 1 2 x
(A) y = | f (x) | (P) O
–1

y
1

x
(B) y = f ( | x | ) (Q) –2 –1 O 1 2
–1
y
1
x
(C) y = f (– | x | ) (R) –2 –1 O 1 2
–1
y
1
1
(D) y = ( | f (x) | – f (x) ) (S) –2
x
2 –1 O 1 2
–1

EXERCISE–II

1. Let f be a oneone function with domain {x,y,z} and range {1,2,3}. It is given that exactly one of the
following statements is true and the remaining two are false.
f(x) = 1 ; f(y)  1 ; f(z)  2. Determine f1(1)

2. Let x = log49 + log928


show that [x] = 3, where [x] denotes the greatest integer less than or equal to x.

3. (a) A function f is defined for all positive integers and satisfies f(1) = 2005 and f(1)+ f(2)+ ... + f(n) = n2f(n)
for all n > 1. Find the value of f(2004).

(b) If a, b are positive real numbers such that a – b = 2, then find the smallest value of the constant L for
which x 2  ax  x 2  bx < L for all x > 0.

(c) Let f (x) = x2 + kx ; k is a real number. The set of values of k for which the equation f (x) = 0 and
f  f (x ) = 0 have same real solution set.

(d) Let P(x) = x6 + ax5 + bx4 + cx3 + dx2 + ex + f be a polynomial such that P(1) = 1 ; P(2) = 2 ; P(3) = 3;
P(4) = 4; P(5) = 5 and P(6) = 6 then find the value of P(7).

(e) Let a and b be real numbers and let f (x) = a sin x + b 3 x + 4,  x  R. If f log10 (log 3 10)  = 5 then
find the value of f log10 (log10 3) .

Get 10% Instant Discount On Unacademy Plus [Use Referral Code: MCSIR] 37
Maths IIT-JEE ‘Best Approach’ (MC SIR) Functions
4. Column I contains functions and column II contains their natural domains. Exactly one entry of column II
matches with exactly one entry of column I.
Column I Column II
 x 2  3x  2 
(A) g (x) = ln  (P) (1, 3)  (3, )
 x  1 

1
(B) h (x) = (Q) (– , 2)
 x 1
ln  
 2 
 1
(C) (x) = ln  x 2  12  2 x  (R)   ,  
   2
(S) [–3, –1)  [1, )

5. Let [x] = the greatest integer less than or equal to x. If all the values of x such that the product
 1  1
x  2  x  2  is prime, belongs to the set [x1, x2)  [x3, x4), find the value of x1  x2  x3  x4 .
2 2 2 2
   

6. Suppose p(x) is a polynomial with integer coefficients. The remainder when p(x) is divided by x – 1 is 1
and the remainder when p(x) is divided by x – 4 is 10. If r (x) is the remainder when p(x) is divided by
(x – 1)(x – 4), find the value of r (2006).
1
 e  | ln{ x }|  {x} | ln{ x }| where ever it exists
7. Prove that the function defined as , f (x) = 

 {x} otherwise , then
f (x) is odd as well as even. (where {x} denotes the fractional part function )

 1    1  x 
8. In a function 2 f(x) + xf    2f  2 sin    x     = 4 cos2 + x cos
 x    4  2 x
Prove that (i) f(2) + f(1/2) = 1 and (ii) f(2) + f(1) = 0

9. A function f , defined for all x , y  R is such that f (1) = 2 ; f (2) = 8


& f (x + y)  k xy = f (x) + 2 y2 , where k is some constant . Find f (x) & show that :
 1 
f (x + y) f   = k for x + y  0.
 x  y

10. Let f : R  R – {3} be a function with the property that there exist T > 0 such that
f (x )  5
f (x + T) = for every x  R. Prove that f (x) is periodic.
f (x)  3

11. If f (x) = 1 + x  2 , 0  x  4
g (x) = 2  x ,  1  x  3
Then find fog (x) & gof (x) . Draw rough sketch of the graphs of fog (x) & gof (x) .

Get 10% Instant Discount On Unacademy Plus [Use Referral Code: MCSIR] 38
Maths IIT-JEE ‘Best Approach’ (MC SIR) Functions

12. Let f (x) = x135 + x125 – x115 + x5 + 1. If f (x) is divided by x3 – x then the remainder is some function
of x say g (x). Find the value of g (10).

13. Let {x} & [x] denote the fractional and integral part of a real number x respectively. Solve 4{x}= x + [x]

9x  1   2   3   2005 
14. Let f (x) = x then find the value of the sum f  +f   +f   + ....+ f  
9 3  2006   2006   2006   2006 

15. Let f (x) = (x + 1)(x + 2)(x + 3)(x + 4) + 5 where x  [–6, 6]. If the range of the function is
[a, b] where a, b  N then find the value of (a + b).

16. Find a formula for a function g (x) satisfying the following conditions
(a) domain of g is (– , ) (b) range of g is [–2, 8]
(c) g has a period  and (d) g (2) = 3

3 4
17. The set of real values of 'x' satisfying the equality   +   = 5 (where [ ] denotes the greatest integer
x x
function) belongs to the interval a , b c where a, b, c  N and b c is in its lowest form. Find the value
of a + b + c + abc.

18. f (x) and g (x) are linear function such that for all x, f  g ( x ) and g  f ( x )  are Identity functions.
If f (0) = 4 and g (5) = 17, compute f (2006).

19. A is a point on the circumference of a circle. Chords AB and AC divide the area of the circle into three
equal parts. If the angle BAC is the root of the equation, f (x) = 0 then find f (x).

20. If for all real values of u & v, 2 f(u) cos v = f (u + v) + f (u  v), prove that, for all real values of x.
(i) f (x) + f ( x) = 2a cos x (ii) f ( x) + f( x) = 0
(iii) f ( x) + f (x) =  2b sin x. Deduce that f (x) = a cos x  b sin x, a, b are arbitrary constants.

21. Given X = {1, 2, 3, 4}, find all oneone, onto mappings, f : X  X such that,
f (1) = 1 , f (2)  2 and f (4)  4. [REE 2000, 3 out of 100]

EXERCISE–III
 4x  3   4x  3 
1. The period of the function f(x) = 4 sin4  2  + 2 cos  2  is :
 6   3 
32 33 4 2 4 3
(A) (B) (C) (D)
4 4 3 3

Get 10% Instant Discount On Unacademy Plus [Use Referral Code: MCSIR] 39
Maths IIT-JEE ‘Best Approach’ (MC SIR) Functions

 1 x 
2. If 2f(x – 1) – f   = x, then f(x) is :
 x 
1 1  (1  x)
(A) 2(1  x)   (B) 2(x – 1) –
3 (1  x)  x

1 1 1 
(C) x2 + +4 (D) (x  2)  
x2 4 (x  2) 

3. If f : R  R be a function satisfying f(2x + 3) + f(2x + 7) = 2,  x  R, then period of f(x) is :


(A) 2 (B) 4 (C) 8 (D) 16

1
4. If 2 < x2 < 3, then the number of positive roots of {x2} =   , (where {x} denotes the fractional
x
part of x) is :
(A) 0 (B) 1 (C) 2 (D) 3

2(e x  e  x )(sin x  tan x)


5. f(x) = is (where [.] denotes the greatest integer function)
 x  2 
2 3
  
(A) an odd function (B) an even function
(C) neither even nor odd (D) both even and odd

6. If f(x + ay, x – ay) = axy, then f(x, y) equals :


x 2  y2 x 2  y2
(A) (B) (C) x2 (D) y2
4 4

7. If f : R  R, f(x) = ex, & g : R  R, g(x) = 3x – 2, then the value of (fog)–1 (x) is equal to
2  log x  x 3
(A) log(x – 2) (B) (C) log   (D) None of these
3  2 

x  [x]
8. Let f(x) = , then range of f(x) is ([.] = G.I.F.) :
1  [x]  x
(A) [0, 1] (B) [0, 1/2] (C) [1/2, 1] (D) [0, 1/2)

9. If f(x) be a polynomial satisfying f(x) . f(1/x) = f(x ) + f(1/x) and f(4) = 65 then f(6) = ?
(A) 176 (B) 217 (C) 289 (D) None of these

Get 10% Instant Discount On Unacademy Plus [Use Referral Code: MCSIR] 40
Maths IIT-JEE ‘Best Approach’ (MC SIR) Functions

10. The range of the function f(x) = sin(cos x)  cos(sin x) is :

(A) 1,1  cos1  (B)  cos1,1  cos1 


   

(C)  cos1,1  sin1  (D) 1,1  sin1 


   
11. Let f(x) = cot(5  3x)(cot(5)  cot(3x))  cot 3x  1 , then domain is :

 n  
(A) R –   , n  I (B) (2n + 1) ,nI
3 6

 n n  5   n  5 
(C) R –  , , n  I (D) R –  nI
3 3   3 
12. If f(x) is even, periodic function defined for all x  R and has period 1, then

 1 1  2 
(A) f  x   = f(x) (B) f   x   f   x 
 2 3  3 
(C) f(x + 1) = f(2x + 1) (D) f(0) can not be zero
13. The number of bijective functions f : A  A, where A = {1, 2, 3, 4} such that f(1)  3, f(2)  1,
f(3)  4, f(4)  2 is :
(A) 11 (B) 23 (C) 12 (D) 9
n(n  1)
14. The period of the function, f(x) = [x] + [2x] + [3x] +.....+ [nx] – x, where n  N and [] denotes
2
the greatest integer function, is :
1
(A) 1 (B) n (C) (D) Non periodic
n
15. Let set A consists of 5 elements and set B consists of 3 elements. Number of functions that can be
defined from A to B which are not surjective is :
(A) 99 (B) 93 (C) 123 (D) None
x 3
16. Let f(x) = , x  –1. Then f2010(2014) [where f n(x) = fof
.....of
 (x)] is :
x 1 n times

(A) 2010 (B) 4020 (C) 4028 (D) 2014


17. Let f(x) = sin x , then
(A) f(x) is periodic with period 2 (B) f(x) is periodic with period 
(C) f(x) is periodic with period 42 (D) None of these
18. Let f be a real valued function such that
 2002 
f (x) + 2 f   = 3x
 x 
for all x > 0. The value of f (2), is
(A) 1000 (B) 2000 (C) 3000 (D) 4000

Get 10% Instant Discount On Unacademy Plus [Use Referral Code: MCSIR] 41
Maths IIT-JEE ‘Best Approach’ (MC SIR) Functions
19. Which one of the following depicts the graph of an odd function?

(A) (B)

(C) (D)

 x   x 
20. The period of the function f(x) = sin 2x + sin    sin   is
 3   5 
(A) 2 (B) 6 (C) 15 (D) 30

8 8 4 4
21. Given f (x) =  and g (x) =  then g(x) is
1 x 1 x f (sin x ) f (cos x )
(A) periodic with period /2 (B) periodic with period 
(C) periodic with period 2 (D) aperiodic

n
k
22. Suppose, f(x, n) =  log x  x  , then the value of x satisfying the equation f(x, 10) = f(x, 11) is
k 1

(A) 9 (B) 10 (C) 11 (D) none

x ln x
23. f(x) = and g(x) = . Then identify the CORRECT statement
ln x x
1 1
(A) and f(x) are identical functions (B) and g(x) are identical functions
g(x) f (x)
1
(C) f(x) . g(x) = 1  x > 0 (D) =1  x>0
f (x) . g(x)

f (x)
24. Let f be a function satisfying f(xy) = for all positive real numbers x and y. If f(30) = 20, then the
y
value of f(40) is :
(A) 15 (B) 20 (C) 40 (D) 60

25. Let f (x) = sin2x + cos4x + 2 and g (x) = cos(cos x) + cos(sin x). Also let period of f (x) and g (x) be
T1 and T2 respectively then
(A) T1 = 2T2 (B) 2T1 = T2 (C) T1 = T2 (D) T1 = 4T2

Get 10% Instant Discount On Unacademy Plus [Use Referral Code: MCSIR] 42
Maths IIT-JEE ‘Best Approach’ (MC SIR) Functions
2
26. Let f (x) = ; g (x) = cos x and h (x) = x  3 then the range of the composite function fogoh, is
x 1
(A) R+ (B) R – {0} (C) [1, ) (D) R+ – {1}

27. If f (x, y) = max( x, y) min( x ,y ) and g (x, y) = max(x, y) – min(x, y), then

  3 
f  g   1,  , g (4,  1.75)  equals
  2 
(A) – 0.5 (B) 0.5 (C) 1 (D) 1.5

28. If the solution set for f (x) < 3 is (0, ) and the solution set for f (x) > – 2 is (– , 5), then the true solution
set for  f (x ) 2  f (x) + 6, is
(A) (– , + ) (B) (– , 0] (C) [0, 5] (D) (– , 0]  [5, )

29. The graph of the function y = g (x) is shown.


1
The number of solutions of the equation g ( x)  1  , is
2
(A) 4 (B) 5
(C) 6 (D) 8

9
30. Let R be the region in the first quadrant bounded by the x and y axis and the graphs of f(x) = x+b
25
and y = f–1(x). If the area of R is 49, then the value of b, is
18 22 28
(A) (B) (C) (D) none
5 5 5

31. Consider the functions


f: XY and g :YZ
then which of the following is/are incorrect ?
(A) If f and g both are injective then gof : X  Z is injective
(B) If f and g both are surjective gof : X  Z is surjective
(C) If gof : X  Z is bijective then f is injective and g is surjective
(D) none

32. Which of the following statements are incorrect ?


I If f(x) and g(x) are one to one then f(x) + g(x) is also one to one
II If f(x) and g(x) are one-one then f(x) . g(x) is also one-one
III If f(x) is odd then it is necessarily one to one
(A) I and II only (B) II and III only (C) III and I only (D) I, II and III

0 if x is rational 0 if x is irrational
 
33. Let f (x) =  and g (x) = 
 x if x is irrational  x if x is rational
Then the function (f – g) x is
(A) odd (B) even
(C) neither odd nor even (D) odd as well as even

Get 10% Instant Discount On Unacademy Plus [Use Referral Code: MCSIR] 43
Maths IIT-JEE ‘Best Approach’ (MC SIR) Functions
34. Let a > 1 be a real number and f(x) = logax2 for x > 0. If f–1 is the inverse function of f and b and c are
real numbers then f–1(b + c) is equal to
1 1
(A) f–1(b) . f–1(c) (B) f–1(b) + f–1(c) (C) (D)
f (b  c) f (b)  f 1 (c)
1

35. Period of f(x) = nx + n – [nx + n], (n  N where [] denotes the greatest integer function is
(A) 1 (B) 1/n (C) n (D) none of these

36. Let f(x) = sin [a] x (where [] denotes the greatest integer function). If f is periodic with fundamental
period , then a belongs to :
(A) [2, 3) (B) {4, 5} (C) [4, 5] (D) [4, 5)

37. Given f (x) is a polynomial function of x, satisfying f(x) . f(y) = f(x) + f(y) + f(xy) – 2 and that f (2) = 5.
Then f (3) is equal to
(A) 10 (B) 24 (C) 15 (D) none

38. The function f is one to one and the sum of all the intercepts of the graph is 5. The sum of all of the graph
of y = f–1(x) is
(A) 5 (B) 1/5 (C) 2/5 (D) – 5

39. The period of the function f(x) = sin(x + 3 – [x + 3]), where [] denotes the greatest integer function is
(A) 2 + 3 (B) 2  (C) 1 (D) 3

40. If f(x) = x2 + bx + c and f(2 + t) = f(2 – t) for all real numbers t, then which of the following is true ?
(A) f(1) < f(2) < f(4) (B) f(2) < f(1) < f(4) (C) f(2) < f(4) < f(1) (D) f(4) < f(2) < f(1)

41. The solution set for [x]{x} = 1 where {x} and [x] are fractional part & integral part of x, is
(A) R+ – (0, 1) (B) R+ – {1}
 1   1 
(C) m  m  I  {0} (D) m  m  N  {1}
 m   m 
42. If f(x) = px + q and f  f  f (x)   = 8x + 21, where p and q are real numbers, then p + q equals
(A) 3 (B) 5 (C) 7 (D) 11

43. If f(x) = 2 tan 3x + 5 1  cos 6x ; g(x) is a function having the same time period as that of f(x), then
which of the following can be g(x).
(A) (sec2 3x + cosec2 3x)tan2 3x (B) 2sin 3x + 3cos 3x
(C) 2 1  cos 2 3x + cosec 3x (D) 3 cosec 3x + 2 tan 3x
8
44. The range of the function y = is
9  x2
8   8 8 
(A) (–, ) – {± 3} (B)  ,   (C)  0,  (D) (–, 0)   ,  
9   9 9 

Get 10% Instant Discount On Unacademy Plus [Use Referral Code: MCSIR] 44
Maths IIT-JEE ‘Best Approach’ (MC SIR) Functions
{x}
45. Range of the function f (x) = where {x} denotes the fractional part function is
1  {x}

1  1  1
(A) [0 , 1) (B) 0,  (C) 0,  (D)  0, 
 2  2  2

46. Given the graphs of the two functions, y = f(x) & y = g(x). In the
adjacent figure from point A on the graph of the function y = f(x)
corresponding to the given value of the independent variable (say x0), a
straight line is drawn parallel to the X-axis to intersect the bisector of
the first and the third quadrants at point B. From the point B a straight
line parallel to the Y-axis is drawn to intersect the graph of the function
y = g(x) at C. Again a straight line is drawn from the point C parallel to
the X-axis, to intersect the line NN ' at D. If the straight line NN ' is
parallel to Y-axis, then the co-ordinates of the point D are
(A) (f(x0), g(f(x0))) (B) (x0, g(x0))
(C) (x0, g(f(x0))) (D) (f(x0), f(g (x0)))

 y y
47. If f  x  , x   = xy then f(m, n) + f(n, m) = 0
 8 8
(A) only when m = n (B) only when m  n
(C) only when m = – n (D) for all m & n

x rx
48. Let f(x) = and let g(x) = . Let S be the set of all real numbers r such that f(g(x)) = g(f (x)) for
1 x 1 x
infinitely many real number x. The number of elements in set S is
(A) 1 (B) 2 (C) 3 (D) 5

49. Let [x] denote the greatest integer in x. Then in the interval [0, 3] the number of solutions of the equation,
x2 – 3x + [x] = 0 is
(A) 6 (B) 4 (C) 2 (D) 0

x3 1
50. The domain of function, satisfying f(x) + f(x–1) = , is
x
(A) An empty set (B) a singleton (C) a finite set (D) an infinite set

51. Let f(x) = ([a]2 – 5[a] + 4)x3 – (6{a}2 – 5{a} + 1) x – (tan x)sgn x, be an even function for all x  R, then
sum of all possible values of ‘a’ is
(where [ ] and { } denote greatest integer function and fractional part functions respectively)
17 53 31 35
(A) (B) (C) (D)
6 6 3 3
k f 4 (k) 2009
52. Let f(k) =
2009
and g(k) =
(1  f (k)) 4  (f (k)) 4
then the sum  g(k) is equal :
k 0
(A) 2009 (B) 2008 (C) 1005 (D) 1004

Get 10% Instant Discount On Unacademy Plus [Use Referral Code: MCSIR] 45
Maths IIT-JEE ‘Best Approach’ (MC SIR) Functions

| sin x |  | cos x |
53. The period of the function f(x) = is
| sin x  cos x |
(A) /2 (B) /4 (C)  (D) 2

54. A function f(x) = 1  2x + x is defined from D1  D2 and is onto. If the set D1 is its complete
domain then the set D2 is
 1
(A)  ,  (B) (–, 2) (C) (–, 1) (D) (–, 1]
 2
55. Which of the following is true for a real valued function y = f (x), defined on [– a, a] ?
(A) f (x) can be expressed as a sum or a difference of two even functions
(B) f (x) can be expressed as a sum or a difference of two odd functions
(C) f (x) can be expressed as a sum or a difference of an odd and an even function
(D) f (x) can never be expressed as a sum or a difference of an odd and an even function
x 1
56. If for all x different from both 1 and 0 we have f1(x) = , f2(x) = , and for all integers n  1,we
x 1 1 x

 f n 1  f1 (x)  if n is odd
have f n+2(x) =  then f4(x) equals
 f n 1  f 2 (x)  if n is even
(A) x (B) x – 1 (C) f1(x) (D) f2(x)

57. Suppose that f(n) is a real valued function whose domain is the set of positive integers and that f(n)
satisfies the following two properties
f(1) = 23 and f(n + 1) = 8 + 3 . f(n), for n  1
It follows that there are constants p, q and r such that f(n) = p . qn – r, for n = 1, 2,.....
then the value of p + q + r is
(A) 16 (B) 17 (C) 20 (D) 26

58. The domain of f(x) = x  2  2 x  3  x  2  2 x  3 is


(A) [3, 5] (B) (3, 5) (C) [5, ) (D) [3, )

e2x 1  1   2   3   2008 
59. If f(x) =
1 e 2x 1 , then the value of f  f  f   + ..... + f  2009  is
 2009   2009   2009   
(A) 1002.5 (B) 1001.5 (C) 1003 (D) 1004

 0 ; x 1
60. f(x) = 
 2x  2 ; x  1
then number of solution(s) to the equation f(f(f(f(x)))) = x
(A) 2 (B) 1 (C) 4 (D) 0

Get 10% Instant Discount On Unacademy Plus [Use Referral Code: MCSIR] 46
Maths IIT-JEE ‘Best Approach’ (MC SIR) Functions
Paragraph for question nos. 61 to 64

Let f (x) = x2 – 2x – 1  x  R. Let f : (– , a]  [b, ), where 'a' is the largest real number for which
f (x) is bijective.
61. The value of (a + b) is equal to
(A) – 2 (B) – 1 (C) 0 (D) 1
62. Let f : R  R, g (x) = f (x) + 3x – 1, then the least value of function y = g(| x |) is
(A) – 9/4 (B) – 5/4 (C) – 2 (D) – 1
63. Let f : [a, )  [b, ), then f –1(x) is given by
(A) 1 + x  2 (B) 1 – x3 (C) 1 – x2 (D) 1 + x3
64. Let f : R  R, then range of values of k for which equation f (| x |) = k has 4 distinct real roots is
(A) (– 2, – 1) (B) (– 2, 0) (C) ( – 1, 0) (D) (0, 1)
Paragraph for question nos. 65 to 67
Define a function  : N  N as follows : (1) = 1, (Pn) = Pn–1(P – 1) if P is prime and n  N and
(mn) =(m) (n) if m & n are relative prime natural numbers.
65. (8n + 4) where n N is equal to
(A) (4n + 2) (B) (2n + 1) (C) 2(2n + 1) (D) 4(2n + 1)
66. The number of natural numbers ‘n’ such that (n) is odd is
(A) 1 (B) 2 (C) 3 (D) none

67. If (7n) = 2058 where n N, then the value of n is


(A) 3 (B) 4 (C) 5 (D) 6

[REASONING TYPE]
68. Consider the function
f(x) = (x+1C2x–8) (2x–8Cx+1)
Statement-1 : Domain of f(x) is singleton.
because
Statement-2 : Range of f(x) is singleton.
(A) Statement-1 is true, Statement-2 is true and Statement-2 is correct explanation for Statement-1.
(B) Statement-1 is true, Statement-2 is true and Statement-2 is NOT the correct explanation for Statement-1.
(C) Statement-1 is true, Statement-2 is false
(D) Statement-1 is false, Statement-2 is true

[MULTIPLE OBJECTIVE TYPE]


69. A continuous function f(x) on R  R satisfies the relation
f(x) + f(2x + y) + 5xy = f(3x – y) + 2x2 + 1 for  x, y  R
then which of the following hold(s) good ?
(A) f is many one (B) f has no minima
(C) f is neither odd nor even (D) f is bounded
70. Which of the following function (s) is/are Transcendental ?
2sin 3x
(A) f(x) = 5 sin x (B) f(x) =
x 2  2x  1
(C) f(x) = x 2  2x  1 (D) f(x) = (x2 + 3) . 2x

Get 10% Instant Discount On Unacademy Plus [Use Referral Code: MCSIR] 47
Maths IIT-JEE ‘Best Approach’ (MC SIR) Functions
71. The functions which are aperiodic are :
(A) y = [x + 1] (B) y = sin x2 (C) y = sin2 x (D) y = sin–1 x
where [x] denotes greatest integer function
72. Which of the following function(s) is/are periodic with period .
(A) f(x) = |sin x| (B) f(x) = [x + ] (C) f(x) = cos(sin x) (D) f(x) = cos2 x
(where [.] denotes the greatest integer function)

73. Which of the following function(s) is/are periodic ?


(A) f(x) = x – [x] (B) g(x) = sin(1/x), x  0 & g(0) = 0
(C) h(x) = x cos x (D) w(x) = sin–1(sin x)

74. Which of the functions defined below are one-one function(s) ?


(A) f(x) = (x + 1), (x  – 1) (B) g(x) = x + (1/x), (x > 0)
2
(C) h(x) = x + 4x – 5, (x > 0) (D) f(x) = e–x, (x  0)
75. Which of the following functions are not homogeneous ?

y xy x  ycos x x  y y x
(A) x + y cos (B) (C) (D) ln  ln  
x xy 2 ysin x  y y  x  x  y 

76. If f(x) is a polynomial function satisfying the condition f(x)  f(1/x) = f(x) + f(1/x) and f(2) = 9 then
(A) 2f(4) = 3f(6) (B) 14f(1) = f(3) (C) 9f(3) = 2f(5) (D) f(10) = f(11)

1  sin x
77. The values of x in [–2, 2], for which the graph of the function y = – sec x and
1  sin x

1  sin x
y= – + sec x, coincide are
1  sin x
 3   3   3     3 
(A)  2,     , 2  (B)   ,     , 
 2   2   2 2 2 2 
     3 
(C)   ,  (D) [–2, 2] –  ,  
 2 2  2 2
78. If the function f(x) = ax + b has its own inverse then the ordered pair (a, b) can be
(A) (1, 0) (B) (–1, 0) (C) (–1, 1) (D) (1, 1)

79. Suppose the domain of the function y = f(x) is –1  x  4 and the range is 1  y  10.
Let g(x) = 4 – 3f(x – 2). If the domain of g(x) is a  x  b and the range of g(x) is c  y  d then which
of the following relations hold good ?
(A) 2a + 4b + c + d = 0 (B) a + b + d = 8
(C) 5b + c + d = 4 (D) a + b + c + d + 18 = 0

80. Suppose f (x) = ax + b and g (x) = bx + a, where a and b are positive integers. If
f  g(50)   g  f (50)  = 28 then the product (ab) can have the value equal to
(A) 12 (B) 48 (C) 180 (D) 210

Get 10% Instant Discount On Unacademy Plus [Use Referral Code: MCSIR] 48
Maths IIT-JEE ‘Best Approach’ (MC SIR) Functions
81. Consider the function f(x) = sgn x and g(x) = x(1 – x2) then which of the follows hold(s) good
(A) (fog)(x) is neither odd nor even
(B) (gof)(x) is even as well as odd
(C) (fog)(x) is neither continuous nor differentiable for some x on (–)
(D) (gof)(x) is continuous and differentiable for every x on (–, )
82. f : R  R ; f(x2 + x + 3) + 2f(x2 – 3x + 5) = 6x2 – 10x + 17  x  R then
(A) f is strictly decreasing (B) f(x) = 0 has a root in (0, 2)
(C) f(x) is an odd function (D) f(x) is invertible

 x 2  2x 3 
log 2 tan x   2
83. The domain of definition of the function, f(x) =  2 tan x   4x  4x 3  where [] denotes the
 

 1 1
greatest integer function is given by the interval  n  , n   where n  I then n can be equal to
 4 2
(A) –5 (B) –1 (C) 0 (D) 1

84.   [a]  x  , where [.] denotes the greatest integer function, has fundamental period  for
f(x) = sin 2

3 5 2 4
(A) a = (B) a = (C) a = (D) a =
2 4 3 5

x 2  sin x cos x x(1  sin x)


85. The number of solutions of the equation = 0 is greater than or equal to
x  cos x x 1
(A) 1 (B) 2 (C) 3 (D) 4

86. Let f(x) = max(1 + sin x, 1, 1 – cos x), x [0, 2] and g(x) = max(1, |x – 1|), x R, then
(A) g(f(0)) = 1 (B) g(f(1)) = 1 (C) f(g(1)) = 1 (D) f(g(0)) = sin 1

4 2
87. Let R = {(x, y) : x, y  R, x2 + y2  25} and R' = {(x, y) : x, y  R, y  x } then
9
(A) domain of R  R' = [–3, 3] (B) Range of R  R' = [0, 4]
(C) Range of R R' = [0, 5] (D) R  R' defines a function

[MATCH THE COLUMN]


1 x 1
88. Let f(x) = x + and g(x) = .
x x2
Match the composite function given in Column-I with their respective domains given in Column-II.
Column-I Column-II
(A) fog (P) R – {–2, –5/3}
(B) gof (Q) R – {–1, 0}
(C) fof (R) R – {0}
(D) gog (S) R – {–2, –1}
 x   2x   y   4y  7x 21
89. The number of ordered pair (x, y) satisfying the equation              y where
 2   3   4   5  6 20
0 < x, y < 30 and [.] denotes greatest integer function is equal to
Get 10% Instant Discount On Unacademy Plus [Use Referral Code: MCSIR] 49
Maths IIT-JEE ‘Best Approach’ (MC SIR) Functions

 x1/3 x 1
90. Let f : R  R f(x) = n  x  x 2  1  and g : R  R, g(x) =  1 x , then the number of real
  2e x 1
solutions of the equation, f–1(x) = g(x) is
91. If the function f(x) = x3 – 9x2 + 24x + c has 3 real and distinct roots ,  and , find the sum of all
possible distinct values of [] + [] + [], where [.] denote greatest integer function.

92. Find the number of values of x of the form 6n ± 1, where n  N, in the domain of the function

64  x 2
f(x) = x ln |x – 1| + .
sin x

93. If N be the numbers of natural numbers less than 2009 which can be expressed in the form of [x[x]] for
some positive real x then sum of the digits of N. ([.] denotes greatest integer function).

94. If the function f : [1, )  [1, ) is defined by f(x) = 2x (x  1), then f1(x) is [JEE '99, 2]
x ( x  1)
 1 1 1
(A)  
 2
(B)
2
1  1  4 log2 x  (C)
2

1  1  4 log2 x  (D) not defined

95. The domain of definition of the function, y (x) given by the equation, 2x + 2y = 2 is
(A) 0 < x  1 (B) 0  x  1 (C)  < x  0 (D)  < x < 1
[JEE 2000 Scr.), 1 out of 35]

 1 , x  0

96. Let g (x) = 1 + x  [ x ] & f (x) =  0 , x  0 . Then for all x , f (g (x)) is equal to
1 , x0

(A) x (B) 1 (C) f (x) (D) g (x)
where [ ] denotes the greatest integer function.

1
97. If f : [1 , )  [2 , ) is given by , f (x) = x + , then f 1 (x) equals
x
x x2  4 x x  x2  4
(A) (B) (C) (D) 1  x2  4
2 1  x2 2
log2 (x  3)
98. The domain of definition of f (x) = is :
x 2  3x  2
(A) R – { 1,  2} (B) ( 2, )
(C) R – { 1,  2,  3} (D) ( 3, ) – { 1,  2}

99. Let E = {1, 2, 3, 4 } & F = {1, 2}. Then the number of onto functions from E to F is
(A) 14 (B) 16 (C) 12 (D) 8
x
100. Let f (x) = , x  1 . Then for what value of  is f (f (x)) = x ?
x 1
(A) 2 (B)  2 (C) 1 (D)  1.
[JEE 2001 (Screening) 5  1 = 5]

Get 10% Instant Discount On Unacademy Plus [Use Referral Code: MCSIR] 50
Maths IIT-JEE ‘Best Approach’ (MC SIR) Functions
101. Suppose f(x) = (x + 1)2 for x  –1. If g(x) is the function whose graph is the reflection of the graph of
f (x) with respect to the line y = x, then g(x) equals
1
(A) – x – 1, x > 0 (B) , x > –1 (C) x  1 , x > –1 (D) x – 1, x > 0
(x  1) 2

102. Let function f : R R be defined by f (x) = 2x + sinx for x  R. Then f is


(A) one to one and onto (B) one to one but NOT onto
(C) onto but NOT one to one (D) neither one to one nor onto
[JEE 2002 (Screening), 3 + 3]
2
x x2
103. Range of the function f (x) = 2 is
x  x 1
 7  7
(A) [1, 2] (B) [1,  ) (C)  2 ,  (D) 1, 
 3  3
x
104. Let f (x) = defined from (0, )  [ 0, ) then by f (x) is
1 x
(A) one- one but not onto (B) one- one and onto
(C) Many one but not onto (D) Many one and onto [JEE 2003 (Scr),3+3]
105. Let f (x) = sin x + cos x, g (x) = x2 – 1. Thus g ( f (x) ) is invertible for x 
          
(A)   , 0 (B)   ,  (C)   ,  (D) 0, 
 2   2   4 4  2
[JEE 2004 (Screening)]

EXERCISE–IV
1.  2
The function f  x   log x  x  1 , is  [JEE Main 2003]
(A) neither an even nor an odd function (B) an even function
(C) an odd function (D) a periodic function
3
2. Domain of definition of the function f (x)  2
 log10 (x 3  x) , is [JEE Main 2003]
4x
(A) (–1, 0)  (1, 2)  (2, ) (B) (a, 2)
(C) (–1, 0)  (a, 2) (D) (1, 2)  (2, )
n
3. If f : R  R satisfies f(x + y) = f(x) + f(y), for all x, y R and f(1) = 7, then  f (r) is
r 1
[JEE Main 2003]
7n(n  1) 7n 7(n  1)
(A) (B) (C) (D) 7n + (n +1)
2 2 2

4. A function f from the set of natural numbers to integers defined by

 n 1
 , when n is odd
f (n)   2 is [JEE Main 2003]
 n
, when n is even
 2one-one nor onto
(A) neither (B) one-one but not onto
(C) onto but not one-one (D) one-one and onto both

Get 10% Instant Discount On Unacademy Plus [Use Referral Code: MCSIR] 51
Maths IIT-JEE ‘Best Approach’ (MC SIR) Functions
5. The range of the function f(x) = 7–xPx–3 is [JEE Main 2004]
(A) {1, 2, 3, 4, 5} (B) {1, 2, 3, 4, 5, 6} (C) {1, 2, 3, 4} (D) {1, 2, 3}

6. If f : R  S, defined by f (x)  sin x  3 cos x  1 , is onto, then the interval of S is


[JEE Main 2004]
(A) [–1, 3] (B) [–1, 1] (C) [0, 1] (D) [0, 3]

7. The graph of the function y = f(x) is symmetrical about the line x = 2, then
[JEE Main 2004]
(A) f(x) = –f(–x) (B) f(2 + x) = f(2 – x)
(C) f(x) = f(–x) (D) f(x + 2) = f(x – 2)

sin 1 (x  3)
8. The domain of the function f (x)  is [JEE Main 2004]
9  x2
(A) [1, 2] (B) [2, 3) (C) [1, 2) (D) [2, 3]

9. A function is matched below against an interval where it is suppose to be increasing. Which of the
following pairs is incorrect matched? [JEE Main 2005]
Inter Function
(A) (–) x3 – 3x2 + 3x + 3
(B) [2, ) 2x3 – 3x2 – 12x + 6
 1
(C)  ,  3x2 – 2x +1
 3
(D) (–, – 4) x3 + 6x2 + 6

10. A real valued function f(x) satisfied the functional equation


f(x – y) = f(x) f(y) – f(a – x) f(a + y)
where a is a given constant and f(0) = 1, f(2a – x) is equal to [JEE Main 2005]
(A) –f(x) (B) f(x) (C) f(a) + f(a – x) (D) f(–x)

11. Let f : N  Y be a function defined as f(x) = 4x + 3 where Y = {y  N : y = 4x + 3 for some x N}.


Show that f is invertible and its inverse is [JEE Main 2008 ]
3y  4 y3 y3 y3
(A) g(y)  (B) g(y)  4  (C) g(y)  (D) g(y) 
3 4 4 4

12. Let f(x) = (x + 1)2 – 1, x  –1


Statement-1 : The set {x : f(x) = f–1 (x) = {0, 1}}
Statement-2 : f is a bijection. [JEE Main 2009]
(A) Statement-1 is true, Statement-2 is true. Statement-2 is not a correct explanation for Statement-1.
(B) Statement-1 is true, Statement-2 is false.
(C) Statement-1 is false, Statement-2 is true.
(D) Statement-1 is true, Statement-2 is true. Statement-2 is not a correct explanation for Statement-1.

Get 10% Instant Discount On Unacademy Plus [Use Referral Code: MCSIR] 52
Maths IIT-JEE ‘Best Approach’ (MC SIR) Functions
13. For real x let f(x) = x3 + 5x + 1, then [JEE Main 2009]
(A) f is onto R but not one-one (B) f is one-one and onto R
(C) f is neither one-one nor onto R (D) f is one-one but not onto R
1
14. The domain of the function f (x)  is [JEE Main 2011]
| x | x
(A) (0, ) (B) (–0) (C) (–) (D) (–)

15. If a  R and the equation – 3(x – [x])2 + 2(x – [x]) + a2 = 0


(where [x] denotes the greatest integer  x) has no integral solution, then all
possible values of 'a 'lie in the interval [IIT Mains 2014]
(A) (– , – 2)  (2, ) (B) (– 1, 0)  (0, 1)
(C) (1, 2) (D) (– 2, – 1)

16. If X = {4n – 3n – 1 : n  N} and Y = {9(n – 1) : n  N}, where N is the set of natural numbers, then
X  Y is equal to : [JEE Main 2014]
(A) Y (B) N (C) Y – X (D) X
1
17. If f(x) + 2f   = 3x, x  0, and S = {x  R : f(x) = f(–x)} ; then S : [IIT Main 2016]
x
(A) is an empty set (B) contains exactly one element
(C) contains exactly two elements (D) contains more than two elements

 1 1 x
18. The function f : R    ,  defined as f(x) = is [JEE Mains 2017]
 2 2 1 x2
(A) invertible (B) injective but not surjective
(C) surjective but not injective (D) neither injective nor surjective

19. Let a, b, c  R. If f(x) = ax2 + bx + c is such that a + b + c = 3 and f(x + y) = f(x) + f(y)+xy,
10
 x, y  R, then  f (n) is equal to [JEE Mains 2017]
n 1

(A) 330 (B) 165 (C) 190 (D) 255

1 1
20. For x R – {0,1}. let f1(x) = , f2(x) = 1 – x and f3(x) = be three given functions. if a function,
x 1 x
J(x) satisfies (f2 J.f1)(x) = f3(x) then J(x) is equal to:
[JEE Main 2019 (09-01-2019-Shift-1)]
1
(A) f1(x) (B) f3(x) (C) f2(x) (D) f3  x 
x

2403 k
21. If the fractional part of the number is , then k is equal to:
15 15
[JEE Main 2019 (09-01-2019-Shift-1)]
(A) 6 (B) 4 (C) 8 (D) 14

Get 10% Instant Discount On Unacademy Plus [Use Referral Code: MCSIR] 53
Maths IIT-JEE ‘Best Approach’ (MC SIR) Functions
22. Let A ={xR : x is not a positive interger}. [JEE Main 2019 (09-01-2019-Shift-2)]
2x
Define a fucntion f: A  R as f(x) = , then f is :
x 1
(A) neither injective nor surjective (B) surjective but not injectivbe
(C) injective but not surjective (D) not injective

23. Let N be the set of natural numbers and two functions f and g be defined as f, g: N  N such that
 n 1 if n is odd
f (n)   n2 and g(n) = n – (–1)n. Then fog is: [JEE Main 2019 (10-01-2019-Shift-2)]
if n iseven
2
(A) both one-one and onto (B) neither one-one nor onto
(C) one-one but not onto. (D) onto but not one-one
x
24. Let f : R R be defined by f(x) = , x R. Then the range of f is
1  x2
[JEE Main 2019 (11-01-2019-Shift-1)]
 1 1  1 1
(A) (–1,1) –{0} (B) R    ,  (C)   ,  (D) R – [–1,1]
 2 2  2 2

1
25. Let a function f :  0,     0,   be defined by f(x) = 1  . Then f is :-
x
[JEE Main 2019 (11-01-2019-Shift-2)]
(A) Injective only (B) Not injective but it is surjective
(C) Both injective as well as surjective (D) Neither injective nor surjective

26. The number of functions f from {1, 2, 3, ...., 20} onto {1, 2, 3, ....., 20} such that f(k) is a multiple of 3,
whenever k is a multiple of 4, is: [JEE Main 2019 (11-01-2019-Shift-2)]
(A) 56 × 15 (B) 65 × (15)! (C) (15)! × 6! (D) 5! × 6!

 1 x   2x 
27. If f(x)  log e   , x  1 , then f  2 
is equal to :
 1 x  1 x 
[JEE Main 2019 (08-04-2019-Shift-1)]
2
(A) (f(x)) (B) 2f(x) (C) -2f(x) (D) 2f(x2)

28. The sum of the solutions of the equation x  2  x  


x  4  2,  x  0  is equal to :

[JEE Main 2019 (08-04-2019-Shift-1)]


(A) 12 (B) 9 (C) 10 (D) 4

29. Let f(x) = ax (a > 0) be written as f(x) = f1(x) + f2 (x), where f1(x) is an even function and f2(x) is an odd
function. Then f1(x + y) + f1(x  y) equals:
[JEE Main 2019 (08-04-2019-Shift-2)]
(A) 2f1(x)f1(y) (B) 2f1(x + y)f2(x – y)
(C) 2f1(x + y)f1(x – y) (D) 2f1(x)f1(y)

Get 10% Instant Discount On Unacademy Plus [Use Referral Code: MCSIR] 54
Maths IIT-JEE ‘Best Approach’ (MC SIR) Functions

x2
30. If the function f : R – {1 , – 1}  A defined by f(x) = , is surjective, then A is equal to :
1 x2
[JEE Main 2019 (09-04-2019-Shift-1)]
(A) [0,) (B) R – {–1} (C) R – [–1,0) (D) R – (–1,0)

10

31. Let  f  a  k   16  2
k 1
10
 1 , where the function f satisfies f(x + y) = f(x)f(y) for all natural numbers x,y

and f(1) = 2. Then the natural number 'a' is : [JEE Main 2019 (09-04-2019-Shift-1)]
(A) 4 (B) 3 (C) 2 (D) 16

1
32. The domain of the defination of the function f  x   2
 log10  x 3  x  is :
4x
[JEE Main 2019 (09-04-2019-Shift-2)]
(A)  2, 1   1, 0    2,   (B)  1, 0   1, 2    3,  
(C)  1, 0   1, 2    2,   (D) 1, 2    2,  

33. Let f(x) = x2, x  R. For any A  R , define g(A) = {x  R : f(x)  A}. If S = [0, 4], then which one of
the following statements is not true? [JEE Main 2019 (10-04-2019-Shift-1)]
(A) g(f(S))  S (B) f(g(S))  f(S)
(C) f(g(S)) = S (D) g(f(S)) = g(S)

sin 2 x  2sin x 5. 1


34. All the pairs (x, y) that satisfy the inequality 2  1 also satisfy the equation:
sin 2 y
4
[JEE Main 2019 (10-04-2019-Shift-1)]
(A) sin x = |sin y| (B) sin x = 2 sin y
(C) 2 sin x = sin y (D) 2|sinx| = 3 sin y

35. Let f(x) = loge(sinx), (0 < x <  ) and g(x) = sin-1 (e-x), (x  0), If  is a positive real number such that
a = (fog)' (  ) and b = (fog) (  ), then: [JEE Main 2019 (10-04-2019-Shift-2)]
(A) a 2  b  a  0 (B) a 2  b  a  2 2
(C) a 2  b  a  0 (D) a 2  b  a  1

 3 1  x2  
36. For x   0,  , let f  x   x , g(x) = tanx and h  x   2
. If   x    hof  og)  x  , then   
 2 1 x  3
is equal to : [JEE Main 2019 (12-04-2019-Shift-1)]
5  7 11
(A) tan (B) tan (C) tan (D) tan
12 12 12 12

37. For x  R , let [x] denote the greatest integer  x, then the sum of the series
 1   1 1   1 2   1 99 
 3    3  100    3  100   ....   3  100  [JEE Main 2019 (12-04-2019-Shift-1)]
       
(A) 133 (B) 135 (C) 131 (D) 153

Get 10% Instant Discount On Unacademy Plus [Use Referral Code: MCSIR] 55
Maths IIT-JEE ‘Best Approach’ (MC SIR) Functions
5
38. If g(x) = x2 + x  1 and (goƒ) (x) = 4x2  10x + 5, then f   is equal to:
4

[JEE Main 2020 (07-01-2020-Shift-1)]


1 3 3 1
(A)  (B) (C)  (D)
2 2 2 2

82x  82x
39. The inverse function of f(x) = , x (–1,1), is
82x  82x
[JEE Main 2020 (08-01-2020-Shift-1)]
1 1 x  1  1 x 
(A)  log8 e  log e   (B) log e  
4  1 x  4  1 x 
1 1 x  1  1 x 
(C)  log8 e  log e   (D) log e  
4 1 x  4 1 x 

x x 
40. Let f : (1,3)  R be a function defined by f  x   , where [x] denotes the greatest integer  x.
1  x2
Then the range of f is : [JEE Main 2020 (08-01-2020-Shift-2)]
 2 1   3 4  2 4 3 4  2 3  3 4 
(A)  ,    ,  (B)  ,  (C)  ,  (D)  ,    , 
 5 2   5 5  5 5 5 5  5 5  4 5 

41. Let S be the set of a ll real roots of the equation, 3x(3x – 1) + 2 = |3x – 1| + |3x – 2|. Then S :
[JEE Main 2020 (08-01-2020-Shift-2)]
(A) is a singleton (B) contains at least four elements.
(C) contains exactly two elements (D) is an empty set.

42. The number of distinct solutions of the equation, log 1 sin x  2  log 1 cos x in the interval [0 , 2]
2 2

is .......
[JEE Main 2020 (09-01-2020-Shift-1)]

43. If A = [x  R : |x| < 2] and


B = {x  R : |x – 2|  3} ; then : [JEE Main 2020 (09-01-2020-Shift-2)]
(A) A  B  [–1, 2) (B) B – A = R – (–2, 5)
(C) A  B  (–2, –1) (D) A  B  R – (2,5)

 | x | 5 
44. The domain of the function f  x   sin 1  2  is ( , a]  [a, ). Then a is equal to :
 x 1 
[JEE Main 2020 (02-09-2020-Shift-1)]

17  1 17 1  17 17
(A) (B) (C) (D) 1
2 2 2 2

Get 10% Instant Discount On Unacademy Plus [Use Referral Code: MCSIR] 56
Maths IIT-JEE ‘Best Approach’ (MC SIR) Functions

45. Let f : R  R be a function which satisfies f (x  y)  f (x)  f (y) x, y  R . If f(1) = 2 and
(n 1)
g(n)   f (k), n  N then the value of n, for which g(n) = 20, is:
k 1
[JEE Main 2020 (02-09-2020-Shift-2)]
(A) 9 (B) 5 (C) 4 (D) 20

46. Let [t] denote the greatest integer  t. Then the equation in x,[x]2 + 2[x + 2]  7 = 0 has :
[JEE Main 2020 (04-09-2020-Shift-1)]
(A) exactly four integral solutions (B) infinitely many solutions
(C) no integral solution (D) exactly two solutions

47. Let A={a,b,c} and B ={1,2,3,4}. Then the number of elements in the set C  {f : A  B | 2  f (A) and
f is not one–one} is_______
[JEE Main 2020 (05-09-2020-Shift-2)]


48. If f(x + y) = f(x) f(y) and  f (x)  2 , x, yN, where N is the set of all natural number, then the value
x 1
f (4)
of is : [JEE Main 2020 (06-09-2020-Shift-1)]
f (2)
2 1 1 4
(A) (B) (C) (D)
3 9 3 9
ax
49. For a suitably chosen real constant a, let a function, f:R–{–a}  R be defined by f (x)  .
ax
 1
Further suppose that for any real number x  a and f  x   a , (fof)(x) = x.Then f    is equal
 2
to: [JEE Main 2020 (06-09-2020-Shift-2)]
1 1
(A) –3 (B) 3 (C) (D) 
3 3

50. Suppose that function f : R  R satisfies f(x + y) = f(x) f(y) for all x, y R and f(1) = 3.
n
If  f (i)  363 , then n is equal to ______
i 1

[JEE Main 2020 (06-09-2020-Shift-2)]


1
x–
51. Let ƒ : R  R be defined as ƒ(x) = 2x – 1 and g : R – {1}  R be defined as g (x)  2.
x –1
Then the composition function ƒ(g(x)) is :
[JEE Main 2021 (24-02-2021-Shift-1)]
(A) onto but not one-one (B) both one-one and onto
(C) one-one but not onto (D) neither one-one nor onto

Get 10% Instant Discount On Unacademy Plus [Use Referral Code: MCSIR] 57
Maths IIT-JEE ‘Best Approach’ (MC SIR) Functions
 1 
52. If a +  = 1, b +  = 2 and af(x)  f    bx  , x  0, then the value of expression
x
  x
 1
f(x)  f  
x
1 is_________. [JEE Main 2021 (24-02-2021-Shift-2)]
x
x

53. Let f, g : N  N such that f(n + 1) = f(n) + f(1)  n  N and g be any arbitrary function. Which of the
following statements is NOT true? [JEE Main 2021 (25-02-2021-Shift-1)]
(A) If fog is one-one, then g is one-one (B) If f is onto, then f(n) = n  n N
(C) f is one-one (D) If g is onto, then fog is one-one

5x
54. A function f(x) is given by f  x   , then the sum of the series
5x  5
 1   2   3   39 
f    f    f    .....  f   is equal to:
 20   20   20   20 
[JEE Main 2021 (25-02-2021-Shift-2)]
19 49 29 39
(A) (B) (C) (D)
2 2 2 2

55. Let x denote the total number of one-one functions from a set A with 3 elements to a set B with 5
elements and y denote the total number of one-one functions from the set A to the set A × B. Then :
[JEE Main 2021 (25-02-2021-Shift-2)]

(A) y = 273x (B) 2y = 91x (C) y = 91x (D) 2y = 273x

k  1 if k is odd
56. Let A = {1, 2, 3, ..., 10} and f : A  A be defined as f (k)  
 k if k is even
Then the number of possible functions g : A  A such that gof = f is
[JEE Main 2021 (26-02-2021-Shift-2)]
5 10
(A) 10 (B) C5 (C) 55 (D) 5!

x2  x  2
57. Let f(x) = sin–1x and g(x)  . If g(2)  lim g(x) , then the domain of the function fog is :
2x 2  x  6 x 2

[JEE Main 2021 (26-02-2021-Shift-2)]


 3 
(A)  , 2    ,   (B)  , 2   1,  
 2 
 4 
(C)  , 2    ,   (D)  , 1   2,  
 3 

58. The number of elements in the set {x   : (|x| – 3) |x + 4| = 6} is equal to :


[JEE Main 2021 (16-03-2021-Shift-1)]
(A) 3 (B) 2 (C) 4 (D) 1

Get 10% Instant Discount On Unacademy Plus [Use Referral Code: MCSIR] 58
Maths IIT-JEE ‘Best Approach’ (MC SIR) Functions
59. The inverse of y = 5log x is : [JEE Main 2021 (17-03-2021-Shift-1)]
1 1
logy log5 log5 log y
(A) x = 5 (B) x = y (C) x  y (D) x  5

cosec 1 x
60. The real valued function f (x)  , where [x] denotes the greatest integer less than or equal to
x  [x]
x, is defined for all x belonging to : [JEE Main 2021 (18-03-2021-Shift-1)]
(A) all reals except integers (B) all non-integers except the interval [–1,1]
(C) all integers except [0,–1,1] (D) all reals except the Interval [–1,1]

61. If the functions are defined as f(x)  x and g(x)  1  x , then what is the common domain of the
following functions : [JEE Main 2021 (18-03-2021-Shift-1)]
f (x)
f + g, f – g, f/g, g/f, g – f where (f ± g) (x) = f (x)  g(x), (f / g)(x) 
g(x)
(A) 0  x  1 (B) 0  x < 1 (C) 0 < x < 1 (D) 0 < x  1

x2
62. Let f : R – {3}  R – {1} be defined by f (x)  . Let g : R  R be given as g(x) = 2x – 3.
x 3
13
Then, the sum of all the values of x for which f 1  x   g 1  x   is equal to
2
[JEE Main 2021 (18-03-2021-Shift-2)]
(A) 7 (B) 2 (C) 5 (D) 3

63. Let [x] denote the greatest integer  x, where x  R. If the domain of the real valued function

x  2 is ( , a)  [b, c)  [4, ), a  b  c, then the value of a + b + c is:


f (x) 
x   3
[JEE Main 2021 (20-07-2021-Shift-1)]
(A) 1 (B) –2 (C) 8 (D) –3

  5x  3
64. Let f : R –    R be defined by f  x   . Then the value of  for which (f0f)(x) = x,
6 6x  
 
for all x  R    , is: [JEE Main 2021 (20-07-2021-Shift-2)]
6
(A) No such  exists (B) 6 (C) 8 (D) 5

65. Let [x] denote the greatest integer less than or equal to x. Then, the value of x  R satisfying the
equation [ex]2 + [ex + 1]–3 = 0 lie in the interval :
[JEE Main 2021 (22-07-2021-Shift-2)]
(a) [0,1/e) (B) [1, e) (C) [loge2, loge­3) (D) [0, loge2)

Get 10% Instant Discount On Unacademy Plus [Use Referral Code: MCSIR] 59
Maths IIT-JEE ‘Best Approach’ (MC SIR) Functions
66. Let A = {0,1,2,3,4,5,6,7} Then the number of bijective functions f : A A such that f(1) + f(2) = 3–f (3)
is equal to-

[JEE Main 2021 (22-07-2021-Shift-2)]

67. Let g : N  N defined as


g(3n + 1) = 3n + 2, g(3n + 2) = 3n + 3, g(3n + 3) = 3n +1,
for all n  0. Then which of the following statements is true?
(A) gogog = g [JEE Main 2021 (25-07-2021-Shift-1)]
(B) There exists a function f : N  N such that gof = f
(C) There exists a one-one function f : N  N such that fog = f
(D) There exists an onto function f : N  N such that fog = f

68. Consider function f : A  B and g : B  C (A, B, C  R) such that (gof)–1 exists, then :
[JEE Main 2021 (25-07-2021-Shift-2)]
(A) f is onto and g is one-one (B) f is one-one and g is onto
(C) f and g both are onto (D) f and g both are one-one

100
 (1)n n 
69. If [x] be the greatest integer less than or equal to x, then   is equal to :
n 8  2 
[JEE Main 2021 (25-07-2021-Shift-2)]
(A) –2 (B) 2 (C) 4 (D) 0

70. Let S = {1, 2, 3, 4, 5, 6, 7}. Then the number of possible function f : S  S such that

f(m.n) = f(m).f(n) for every m, n  S and m·n  S is equal to ______.


[JEE Main 2021 (27-07-2021-Shift-1)]

1
71. Let f : R  R be defined as f(x + y) + f(x – y) = 2f(x) f(y), f   = –1.
 2
20
1
Then the value of  is equal to :
k 1 sin  k  sin  k  f (k) 
[JEE Main 2021 (27-07-2021-Shift-2)]
2
(A) sec (21) sin (20) sin (2) (B) sec2 (1) sec (21) cos (20)
(C) cosec2 (1) cosec (21) sin (20) (D) cosec2 (21) cos (20) cos (2)

1 x 
72. The domain of the function cos ec 1   is:
 x 
[JEE Main 2021 (26-08-2021-Shift-2)]
 1   1   1  1 
(A)   ,    0 (B)   ,    0 (C)  1,     0,   (D)   , 0   1,  
 2   2   2  2 

Get 10% Instant Discount On Unacademy Plus [Use Referral Code: MCSIR] 60
Maths IIT-JEE ‘Best Approach’ (MC SIR) Functions

73.  
If S  x  R : x  2  1 , B  x  R : x 2  3  1 , C  x  R : x  4  2 and Z is the set of all
C
integers, then the number of subsets of the set  A  B  C   Z is ______.
[JEE Main 2021 (27-08-2021-Shift-1)]

74. Which of the following is not correct for relation R on the set of real numbers?
[Jee main 2021 (31-08-2021-shift-1)]

(A)  x, y   R  x  y  1 is reflexive and symmetric

(B)  x, y   R  0  x  y  1 is neither transitive nor symmetric

(C)  x, y   R  x  y  1 is reflexive but not symmetric

(D)  x, y   R  0  x  y  1 is symmetric and transitive

75. Let f : N  N be a function such that f  m  n   f  m   f  n  for every m, n  N . If f(6) = 18 then


f  2   f  3 is equal to: [JEE Main 2021 (31-08-2021-Shift-2)]
(A) 18 (B) 36 (C) 6 (D) 54

76. The range of the function


  3       3 
f  x   log 5  3  cos   x   cos   x   cos   x   cos   x   is
  4  4  4   4 
[JEE Main 2021 (01-09-2021-Shift-2)]
 1 
(A)  , 5 
 5 

(B) 0, 5  (C)  0, 2 (D)  2, 2

2
77. Let f(x) be a polynomial of degree 3 such that f  k    for k = 2, 3, 4, 5. Then the value of
k

52 – 10 f(10) is equal to:


[JEE Main 2021 (01-09-2021-Shift-2)]

EXERCISE–V
1. Let f(x) = x2 and g(x) = sin x for all x  R. Then the set of all x satisfying (f o g o g o f)(x) = (g o g o f)(x),
where (f o g)(x) = f(g(x)), is [JEE Adv. 2011]
(A) ± n , n  {0, 1, 2, ....} (B) ± n , n  {1, 2, ....}

(C) + 2n, n  {...., –2, –1, 0, 1, 2.....} (D) 2n, n  {...., –2, –1, 0, 1, 2.....}
2

2. The function f : [0, 3] [1, 29], defined by f(x) = 2x3 – 15x2 + 36x + 1, is [JEE Adv. 2012]
(A) one-one and onto (B) onto but not one-one
(C) one-one but not onto (D) neither one-one nor onto

Get 10% Instant Discount On Unacademy Plus [Use Referral Code: MCSIR] 61
Maths IIT-JEE ‘Best Approach’ (MC SIR) Functions

  
3. Let f :   ,   R be given by f(x) = (log(sec x + tan x))3 [JEE Adv. 2014]
 2 2
Then
(A) f(x) is an odd function (B) f(x) is a one-one function
(C) f(x) is an onto function (D) f(x) is an even function

4. Let X be a set with exactly 5 elements and Y be a set with exactly 7 elements. If a is the
number of one-one functions from X to Y and b is the number of onto functions from Y to X,
1
then the value of (b – a) is. [JEE Adv. 2018]
5!

5. If the function f : R R is defined by f(x) = |x| (x–sinx), then which of the following statements is TRUE?
[JEE Adv. 2020]
(A) f is one-one, but NOT onto (B) f is onto, but NOT one-one
(C) f is BOTH one-one and onto (D) f is NEITHER one-one NOR onto

4x
6. Let the function f :[0,1]   be defined by f ( x )  Then the value of
4x  2
 1   2   3   39  1
f   f   f     f   f   is [JEE Adv. 2020]
 40   40   40   40   2

Get 10% Instant Discount On Unacademy Plus [Use Referral Code: MCSIR] 62
Maths IIT-JEE ‘Best Approach’ (MC SIR) Functions
ANSWER KEY
CONCEPT BUILDING-01
1. D
 5 3       3 5   1
2. (i)   4 , 4     4 , 4    4 , 4  (ii)  4,  2   (2, ) (iii) (–, –3]
       

1   1 1 
(iv) (–, –1)  [0, ) (v) (3 – 2 < x < 3 – ) (3 < x  4) (vi)  0,   , 
 100   100 10 

1  5  1  5 
(vii) (–1 < x < –1/2)  (x > 1) (viii)  ,0   

, 
 (ix) (–3, –1] {0}  [1, 3)
 2   2 
(x) {4} [5, ) (xi) (0, 1/4) (3/4, 1) {x : x N, x  2}
1  5  1 
(xii)   ,    , 6  (xiii) [–3, –2) [3, 4) (xiv) R –  2 ,0
 6 3  3   
(xv) 2K < x < (2K + 1) but x  1 where K is non-negative integer
3. (i) D : x  R ; R : [0, 2] (ii) D : x  R ; R : [–1, 1]
(iii) D : {x | x R ; x – 3 ; x 2};R : {f(x) | f(x) R, f(x) 1/5 ; f(x) 1}
(iv) D : R ; R : (–1, 1) (v) D : x R ; R : [5, 6)
(vi) D : x (–1, 5) ; R : (–, 2]
4. B
5. D
6. (i) (a) 2K x  2K +  where K I (b) [–3/2, –1]
(ii) (a) Range : [–1/3, 3], Domain = [4, 7] ; (b) Range [–1, 9] and domain [11, 14]

CONCEPT BUILDING-02
1. (a) Neither injective nor surjective (b) Neither injective nor surjective
(c) Both injective and surjective (d) Neither injective nor surjective
2. B 3. A 4. D 5. A 6. B
7. C 8. B 9. C
ex  ex log 2x 1 1 x
10. (a) ; (b) ; (c) log
2 log 2x  1 2 1 x

CONCEPT BUILDING-03
1. (a) odd, (b) even (c) neither odd nor even (d) odd
(e) neither odd nor even
2. A 3. (i) B (ii) A (iii) B 4. C
5. 64 6. 30 7. 102 8. B
x
9. (a) y = log (10 – 10 ), –  < x < 1 (b) y = x/3 when – < x < 0 & y = x when 0  x < + 
10. (i) period of fog is , period of gof is 2 ; (ii) range of fog is [–1, 1], range of gof is [– tan 1, tan 1]

Get 10% Instant Discount On Unacademy Plus [Use Referral Code: MCSIR] 63
Maths IIT-JEE ‘Best Approach’ (MC SIR) Functions
ASSIGNMENT (prilepko)
Answer key
1. D(y)  [0, 2] 2. D(y)  [ 1,1]
3. D(y)  [1, 6] 4. D(y)  ( , 2]  [3, )
5. D(y)  [ 3,5) 6. D(f )  [ 1, 2]
7. D(y)  [ 1 / 2,3 / 2] 8. D(y)  [ 2 / 3,3]
9. D(y)  [ 2,1)  (1, ) 10. D(y)  ( , 1/ 2)  (3,  )
11. D(f )  ( , 2]  [0, 2] 12. D(f )  ( ,  3]  [0, 3]
13. D(y)  ( ,1)  (1, ) 14. D(y)  ( 4,1]
15. D(y)  [7 / 3, 63)  (63,  ) 16. ( 3, 0)  (0, 2)  (2, 4]
17. D(y)  ( , 0)  [2,3] 18. D(f )  ( , 4]  [5, 6]
19. D(f )  ( , 3]  (2,  ) 20. D(y)  ( 3,3)  (3, 4]
21. D(y)  [ 2,1)  (1, 2] 22. D(y)  ( 3,1]
23. D(y)  ( ,3 / 4)  (4, 7] 24. D(y)  ( , 1)  (4,  )
25. D(y)  ( , 4)  ( 2, 2]  (2,  ) 26. D(y)  {1}
27. D(y)  ( , 4]  (7,  ) 28. D(y)  [5, 7)
29. D(y)  [ 2, 1)  (1,1)  (1,3] 30. D(f )  ( ,  ]
31. D(y)  ( , 0] 32. D(f )  {[ n, (2n  1) / 2] | n  z}
33. D(y)  ( 1/ 6,  / 3]  [5 / 3, 6)
  5   5 13  
34. D(y)    2n,  2n     2n;   2 n  | n  z 
 6 6   6 6  
35. D(y)  (3,30)  (30,  ) 36. D(y)  [ 5,8)  (8,9)
37. D(f )  [4,11 / 2)  (11 / 2, ) 38. D(y)  ( , 2)  (2,  ]
39. D(y)  ( , 3)  ( 1,  ) 40. D(y)  [0,  )
41. D(y)  ( , 7)  ( 1,  ) 42. D(y)  ( 1,1)
43. D(y)  [ 1,1) 44. D(y)  ( 5, 0)  (3,  )
45. D(y)  (2, 4] 46. D(y)  [1,  )
47. D(f )  (2,  ) 48. D(y)  [1,  )
49. D(y)  (0,1) 50. D(y)  (0,3 / 2)
51. D(y)  (–3, –2 / 3) 52. D(f )  [0,  )
53. D(y)  (– , –11)  ( 11, 10)  ( 10, 9)  (9, 2]  [3,  )
54. D(y)  (9, –  ) 55. D(y)  [1, 4]
56. D(y)  {4}  [5,  ) 57. D(f )  ( 2,  3)  ( 3, 2]
58. D(y)  ( 3, 2 / 5)  (2,  ) 59. D(y)  ( , 1)  [0,  )
60. D(y)  (1, 4] 61. D(y)  (1, 4)
62. D(y)  (1,  ) 63. D(y)  (0,1)
64. D(y)  [2,3] 65. D(y)  [ 5, 3]  (3,5)

Get 10% Instant Discount On Unacademy Plus [Use Referral Code: MCSIR] 64
Maths IIT-JEE ‘Best Approach’ (MC SIR) Functions
66. D(y)  (1, 6)  (6,  )
67. D(f )  ( 2, (1  21) / 2]  [1  21) / 2, 3) 68. D(y)  [2, 4) 69. D(f )  (, 2)
70. D(y)  [(1  5) / 2, 0)  [(1  5) / 2,  ) 71. D(f )  ( , 7]  ( 3, )
72. D(y)  (8 / 3,  ) 73. D(f )  ( , 2]  (1, 2]
74. D(y)  [ 1,1)  (3,5] 75. D(f )  [0,1)
76. D(f )  ( ; (1  65) / 4  [0, (1  65) / 4) 77. D(f )  (0,16]
78. D(y)  [1, 2)  (2,5] 79. D(y)  ( , 2)  (3,  )
80. D(y)  (1,3] 81. D(y)  [3,  )
82. D(f )  (2,  ) 83. D(f )  (5,  )
84. D(f )  [3,  ) 85. D(y)  ( , 1)  (1, 4)
86. D(y)  (1, 2)  (2,3] 87. D(y)  ( , 1/ 2]  (0,1)
88. D(y)  [4,5)  (5, 39) 89. D(y)  (2,3)
90. D(y)  (1/ 3, ) 91. D(y)  [103 ,104 )
92. D(y)  [4  2,3)  [4  2, ) 93. D(y)  (100,  )
94. D(y)  (0,10 2 )  (102 ,101/ 2 ) 95. D(y)  (0,1)
96. D(y)  (  , 5)  ( 5, 4)  (4, 5)  (5,  )
97. D(y)  [ 4, ]  [0, ] 98. D(y)  (0,10 2 )  (103 , )
99. D(y)  ( , 5 / 3)  (1,  ) 100. D(y)  (3  2, 3  )  (3, 4]
101. D(y)  (9, ) 102. D(y)  [1, 4)
103. D(y)  [ 1,3]
104. D(y)  ( 1 / 2,1/ 2)  (1 / 2,1)  (3 / 2, )
105. D(y)  (0,102 )  (10 2 ,101 ) 106. D(y)  {1 / 2}
107. D(y)  {[2n, (2n  1)] | n  z}
108. D(y)  {[2n  arcsin(1 / 3); (2n  1)  arcsin(1/ 3)] | n  z}
109. D(y)  {[4n, 2(2n  1)] | n  z}
110. D(y)  {( (2n  1)  arccos(1 / 4), (2n  1)  arccos(1 / 4)) | n  z}
111. D(y)  {[ (6n  1) / 3, (6n  1) / 3] | n  z}
112. D(y)  {(4n  1) / 2 | n  z}  {[ (2m  1), 2(m  1)] | m  z}
113. D(y)  ( , 0)  (0,  ), E(y)  {1,1}
114. D(f )  [0,1], E(f )  [0,1 / 2]
115. D(y)  ( ,  ), E(y)  [ 11 / 3, )
116. D(y)  ( ,  ), E(y)  [log(11/ 3); )
117. D(y)  ( ,  ), E(y)  [log(4 / 5), )
118. D(f )  [1,3], E(f )  [ 2, 10]
119. D(f )  ( ,  ), E(f )  [1, 2]
120. D(f )  [ 1, 2], E(f )  [ 3, 6]

Get 10% Instant Discount On Unacademy Plus [Use Referral Code: MCSIR] 65
Maths IIT-JEE ‘Best Approach’ (MC SIR) Functions
EXERCISE–I
1. (i) {x 1000  x < 10000} (ii) (–2, –1) U (–1, 0) U (1, 2) (iii) (1, 2)  (2, 5/2);

3
(iv) x  {4, 5} (v) x  (3, 5) {x  , }
2

2. (i) D : 1  x  2 R :  3, 6 
(ii) 
D : x  (2n, (2n + 1))  2 n  6 , 2 n  2 , 2 n  56 , n  I and 
R : loga 2 ; a  (0, )  {1}  Range is (–, ) – {0}
 1   1 1
(iii) D : [– 4, ) – {5}; R :  0,    , 
 6   6 3
3. (b) (i), (iii) are identical

4. (a) neither surjective nor injective (b) surjective but not injective
(c) neither injective nor surjective
5. (a) – 3/4; (b) 64; (c) 30, (d) 102; (e) 5050; (f) 28
6. (a) domain is x  0 ; range [–1, 1]; (b) domain 2k  x  2k + ; range [0, 1]
(c) Domain x  R; range [– sin 1, sin 1]; (d) domain is 0  x  1; range is [0, 1]
7. 1
( x  3)10 1024
9. (a) 10 , domain is R, ; (b) f(x) = x2; g (x) = cos x; h (x) = x + 9
( x  3)  1 1025
1
if 0x1
 x2
10. g( x )   11. (a) {–1, 1} (b) a  {0, – 4}
 2
 x if x 1

x if x0 x2 if x0
  x 2 if 0  x  1  1 x if 0  x  1
12. (gof)(x) =  ; (fog)(x) = 
 1  x 2 if x 1  x if x 1
13. (a) even, (b) even, (c) even (d) even
14. (i) (a) y = log (10  10x) ,   < x < 1
(b) y = x/3 when   < x < 0 & y = x when 0  x < + 
(ii) (a) 2K  x  2K +  where K I (b) [3/2 , 1]
(iii) (a) Range : [– 1/3, 3], Domain = [4, 7] ; (b) Range [–1, 9] and domain [11, 14]

15. x = 10; f–1(x) = 10 log 2 ( x 8)


16. (a) 0.86 (b) ± 1, ± 3, ± 5, ± 15
17. (A) R; (B) S; (C) P; (D) Q 18. (A) S; (B) R; (C) P; (D) Q

Get 10% Instant Discount On Unacademy Plus [Use Referral Code: MCSIR] 66
Maths IIT-JEE ‘Best Approach’ (MC SIR) Functions
EXERCISE–II
1. f1(1) = y 2. 152
1
3. (a) , (b) 1, (c) [0, 4), (d) 727, (e) 3 4. (A) S; (B) P; (C) Q
1002
5. 11 6. 6016 9. f (x) = 2 x2
x 1 , 0  x  1
 (1  x) ,  1  x  0 3 x , 1  x  2
11. fog (x) = ; gof (x) = ;
x1 , 0x2 x 1 , 2  x  3
5x , 3  x  4

x , 1  x  0
x , 0 x 1
fof (x) = 4  x , 3  x  4 ; gog (x) = x , 0x2
4x , 2  x  3
12. 21 13. x = 0 or 5/3
14. 1002.5 15. 5049 16. g (x) = 3 + 5 sin(n + 2x – 4), n  I

17. 20 18. 122 19. f (x) = sin x + x 
3
21. {(1, 1), (2, 3), (3, 4), (4, 2)} ; {(1, 1), (2, 4), (3, 2), (4, 3)} and {(1, 1), (2, 4), (3, 3), (4, 2)}

EXERCISE–III
1. B 2. A 3. C 4. B 5. A
6. B 7. B 8. D 9. B 10. C
11. B 12. B 13. D 14. A 15. B
16. D 17. D 18. B 19. D 20. D
21. A 22. C 23. A 24. A 25. C
26. C 27. D 28. D 29. D 30. C
31. D 32. D 33. A 34. A 35. B
36. D 37. A 38. A 39. C 40. B
41. D 42. B 43. A 44. D 45. C
46. C 47. D 48. B 49. C 50. C
51. D 52. C 53. C 54. D 55. C
56. C 57. A 58. D 59. D 60. A
61. B 62. C 63. A 64. A 65. C
66. B 67. B 68. B 69. AB 70. ABD
71. ABD 72. ACD 73. AD 74. ACD 75. BC
76. BC 77. AC 78. ABC 79. BD 80. AD
81. BCD 82. BD 83. AC 84. AB 85. ABC
86. AB 87. AC 88. (A)  S; (B)  Q; (C)  R; (D)  P
89. 4 90. 4 91. 15 92. 2 93. 18(990)
94. B 95. D 96. B 97. A 98. D
99. A 100. D 101. D 102. A 103. D
104. A 105. C

Get 10% Instant Discount On Unacademy Plus [Use Referral Code: MCSIR] 67
Maths IIT-JEE ‘Best Approach’ (MC SIR) Functions

EXERCISE–IV
1. C 2. A 3. A 4. D
5. D 6. A 7. B 8. B
9. C 10. A 11. D 12. B
13. B 14. B 15. B 16. A
17. C 18. C 19. A 20. B
21. C 22. C 23. D 24. C
25. Bonus 26. C 27. B 28. C
29. A 30. C 31. B 32. C
33. D 34. A 35. D 36. D
37. A 38. A 39. A 40. A
41. A 42. 8 43. B 44. C
45. B 46. B 47. 19 48. D
49. B 50. 5 51. C 52. 2
53. D 54. D 55. B 56. A
57. C 58. B 59. C 60. B
61. C 62. C 63. B 64. D
65. D 66. 720 67. D 68. B
69. C 70. 490 71. C 72. B
73. 256 74. D 75. D 76. C
77. 26

EXERCISE–V
1. A 2. B 3. A, B, C 4. 119
5. C 6. 19.00

Get 10% Instant Discount On Unacademy Plus [Use Referral Code: MCSIR] 68
Maths IIT-JEE ‘Best Approach’ (MC SIR) Functions
REVISION PLANNER FOR MAINS
CONCEPT BUILDING 01 :
2. (iii) 5.
CONCEPT BUILDING 03 :
5.
EXERCISE–I
3. (b) 7.
EXERCISE–II
14.
EXERCISE–III
7. 9. 10. 12. 16. 21. 26.
31. 38. 42. 51. 54. 58. 99.
100.
EXERCISE–IV
4. 5. 7. 12. 26. 28. 29.
34. 35.
EXERCISE–V
3.
REVISION PLANNER FOR ADVANCED
CONCEPT BUILDING 03 :
6. 7.
EXERCISE–I
5. 11. 18.

13. (b) 15.


EXERCISE–II
1. 3. (c) (d) 10. 18. 21.

EXERCISE–III
5. 15. 18. 28. 30. 34. 40.
48. 52. 56. 60. 64. 77. 79.
81. 86. 90. 91. 94. 101. 105.
EXERCISE–IV
22. 26.
EXERCISE–V
1. 4.

Get 10% Instant Discount On Unacademy Plus [Use Referral Code: MCSIR] 69

You might also like